SlideShare a Scribd company logo
1 of 26
Download to read offline
FINAL COACHING QUESTIONS IN GENERAL EDUCATION-200 ITEMS
BY
PROF. CRISENCIO M. PANER
(LET TOPNOTCHER & VETERAN REVIEWER)
1. Cris Paner answered correctly 3/5 of the test. How many percent did she miss?
A. 60% C. 30%
B. 50% D. 40%
2.An angle is 14° more than its complement. Find the two angles.
A. 45°& 45° C. 30°& 60°
B. 52°& 38° D. 55°& 125°
3. Evaluate 30
+ 3-2
( 34
)
A. 81 C. 10
B. 9 D. 27
4. A Gynecologist uses 1 ½ hours to treat a patient. She usually works 12 hours a day, from Monday to
Saturday. How many patients can she treat in one week?
A. 16 C. 48
B. 8 D. 36
5. If 15 is subtracted from three times a certain number the result is 27. Find the number.
A. 14 C. 81
B. 9 D. 20
Situational
Maraming katulad ang buhay ng tao. Ito’y tulad ng putahe na “sweet sour dish.” May matamis at
maasim na lasa. Katulad din ito ng panahon, may mabagyo at may maaraw. Nakararanas tayo ng
malulungkot na pangyayari at kung minsan ay masasayang karanasan.
Kainlangan ay lagi tayong handa. Anuman ang pangyayaring dumating ay dapat nating harapin ng buong
tapang, maging ito’y mapait na karanasan. Kung mabagyo ang buhay, lagi nating sabihin ang ganito:
Kung kaya nila itong nararanasan ko, kakayanin ko rin. Hindi ito ibibigay ng Panginoon kung hindi ko
kaya. Nagtitiwala ako sa kanta dahil hindi Niya ako pinapabayaan.
6. Ano ang mahalagang pag-uugali ng tao ang pinahihiwatig ng binasang talataan?
A. Maging makatarungan tayo sa lahat ng bagay, tao at panahon.
B. Humanda tayong lagi sa mga suliraning darating sa ating buhay.
C. Maging positibo tayo sa pag-iisip at pagbibigay pasiya.
D. Gawin natin lagi ang magagandang bagay, ang pagtawag at paghingi ng tulong sa Diyos.
7. Saan itinulad ng may-akda an gating buhay?
A. Sa malakas na bagyo
B. Sa masarap na putahe
C. Sa init at lamig ng panahon
D. Sa putahe at sa panahon
8. Ano ang pangunahing paksa na inilahad sa talataan?
A. Harapin ang buhay at isaisip ang kasabihang umasa sa Diyos na hindi magpapabaya.
B. Pagkat kaloob ng Diyos sa tao ang pagdurusa, sakit at suliranin, nariyan Siya upang tao ay damayan.
C. Harapin nang buong tapang ang buhay na may lungkot at ligaya, at magtiwala sa Poong Maykapal.
D. Buong giting na humarap sa buhay mabagyo man o maaraw.
9. Ano ang nagbunsod ng pagbabago sa yugto ng panitikang Filipino?
A.Pagdating ng Amerikano sa bansa
B.Pag-aabuso ng kastilasa mamamayan
C.Pag-alis ng mga tao da kawit
D.Panggunita kina Padre Gomez, Burgos at Zamora
10.Lack of circulation and oxygen caused in child to die of:
A.asphyxiation C. pneumonia
B.heart failure D. suffocation
11. Fish in aquarium all died when a drop of ____ was placed in the water.
A.alcohol C.sulphur
B.soap D.sugar
12. Children with poor eye sights were advised to improve their nutrition through this were advised to
improved their nutrition through this combination of food:
A.pork, bread, potato
B.rice, squash, fish
C.meat, rice, ice cream
D.hamburger, french fries, coke
13. Water is sold in bottles. The best purifying process is done through:
A.Osmosis C. Boiling
B.Purification D. Distillation
14.Children in upland places are found affected by enlarged goiter. Which of these situations may help
these group of children?
A.Add iodized salt to diet
B.Add iron to diet
C.Expose the children to ultraviolet rays
D.Add iodine drops to diet
15.The urbanization of municipalities have added delays in the Agrarian Reform efforts, because _____.
A.Limited land area resulted in converting public land to subdivisions
B.Agricultural lands were converted to residential areas
C.Coordination of DAR, DA, AND DILG is met
D.Agricultural lands were sold to industrial establishments
16.The appointment of cabinet members was recalled because ___________.
A.The official had no clearance
B.The resistance was strong
C.The official was incompetent
D.The appointing officer was mislead
17.Rural families do not believe in limiting the number of children since they argue:
A.A small family is not ideal for farmers who own animals and work on wide track of land
B.A big family is always happy and economically advantageous
C.Having more children means more helping hands
D.God does the limiting of children not man
18. A young girl left her baby at the household of a monastery. The child grew up without knowing her
mother and father. What right is deprived of the child?
A.Right to assert individuality
B.Right to freedom of choice
C.Right to life
D.Right to quality education
19.No country will progress unless he is a part of a world organization and its agreement. What is
referred to?
A.UNESCO B.NATO C.WHO D.GATT
20. Which is the BEST WAY to write the underlined portion of this sentence?
However, their VCR kept them from missing their favorite primetime shows.
A. keepes
B. keeps
C. hard kept
D. keeped
21. Which is the BEST WAY to write the underlined portion of this sentence?
A person should keep in mind some basic safety rules when you are deciding whether or not to use a fire
extinguisher.
A. rules you decide
B. rules when you decided
C. rules you are deciding
D. rules when deciding
22. What correction should be made to this sentence?
Most State tourism departments and some travel agencies have bed and breakfast listings.
A. Insert a comma after agencies
B. Change have to has
C. Change tourism to Tourism
D. Change State to state
23.What correction should be made to this sentence?
Recently, educators exammined the effectiveness of computer instruction in schools.
A. Replace educators with educator's
B. Change the spelling of exammined to examined
C. Change schools to Schools
D. Replace computer with computer's
24. Which is the BEST WAY to write the underlined portion of this sentence?
However, if a fire extinguisher is handy, a quick-thinking person often can use them to put out a small
fire.
A. They
B. Him
C. Them
D. lt
25.Anong dulog pampanitikan ang kilala rin sa tawag bilang reader-response theory?
A. Antropolohiya
B. Impresyonista
C. Patalambuhay
D. Pansikolohiya
26. Ano ang tinataglay ng mga sumusunod na salita: tanaw, aliw, kamay, reyna?
A. Diptonggo
B. Pares minimal
C. Klaster
D. Ponema
27. What do you call the molecule that contains the genetic information of the organism?
A. Nucleic Acid
B. Nucleolus
C. Ribosomes
D. Organelles
28. Which power of the state enables it to impose charge of burden upon persons, property or property
rights for the use and support of the government expenditures for social services and a way of revenue
collection?
A. Eminent domain
B. Expropriation
C. Value added tax
D. Taxation
29. What was the first term given by Marcelo H. Del Pilar to the notorious invisible influence and
domination by Spanish religious priests over the colonial government.
A. Lassuert partidas
B. Pase Region
C. Frailo cracia
D. Complace
30. The fundamental right invoked by filing the “Writ of Amparo” is _____
A. The right to self defense
B. The right to due process
C. The right to life, liberty and security
D. The right to be defended by a public attorney
31. Which of the following is considered the lowest form of learning?
A. Perceiving
B. Responding
C. Conditioning
D. Teaching
32. Reason must be used in understanding the existence of God. Who advocated this philosophy?
A. St. Benedict
B. St. Peter
C. St. John
D. St. Thomas Aquinas
33. If the principles and theories of human behavior were to be applied to teaching and learning. The
field will be called ______.
A. Educational Theory
B. Educational Philosophy
C. Educational Psychology
D. Educational Sociology
34. In July 1901, Isabelo delos Reyes founded the first labor union in the country. What was its name?
A. Association of the Philippine Labor
B. Union Obrera Democratica
C. Union Trabajadores de Filipinos
D. Association De Compania Tabacalera
35. The Philippine lies in the _____, an area where volcanoes are active.
A. Archipelagic fault line
B. Ring of fire
C. Wheel of fire
D. Volcanic Rim
36. During the June 12, 1898 Declaration of Independence, a band played the Marcha Nacional Filipino
What band was this?
A. Pangkatkawayang ng Pateros
B. Kawit Cavite Band
C. Malabon Band
D. San Francisco de Malabon
37. This is modern technology’s response to message previously sent over couriers or post offices
A. E-shopping
B. E-Registry
C. E-Mail
D. Frailocracia
38. What is the case study method of research?
A. An intensive study of one individual. Typically, this may involve interviews, observation, experiments
and tests.
B. It involves the acquisition, synthesis, organization and presentation of information.
C. The research manipulates variables, randomly assign participants to various conditions and seek to
control other influences.
D. Involves going out and asking a questions about the phenomenon of interest.
39. What is the topical method of research?
A. An intensive study of one individual. Typically, this may involve interviews, observation, experiments
and tests
B. It involves the acquisition, synthesis, organization and presentation of information.
C. The research manipulates variables, randomly assign participants to various conditions and seek to
control other influences.
D. Involves going out and asking a questions about the phenomenon of interest.
40. Not many students are aware that Jose Rizal, the Philippine's National Hero is also a sculptor. Which
among the following is a sculpture of Rizal?
A. The Oblation
B. The Power of Science over Death
C. Lapu-Lapu Monument
D. The Philippine Map Relief
41. Teacher Cris Paner, BSED graduate resigns in her current teaching position as high school teacher
because she has been force to teach in preschool department by the Principal in this school year.
Teacher Jenny's voluntary resignation resulted to?
A. Structural Unemployment
B. Cyclical Unemployment
C. Frictional Unemployment
D. Unemployment
42. The Comprehensive Agrarian Reform Law popular as CARL is the lawful basis of the full
Implementation of the Comprehensive Agrarian Reform Program (CARP). This was during the
administration of?
A. Fidel V. Ramos
B. Ferdinand Marcos
C. Corazon Aquino
D. Joseph Estrada
43. How is coral atool formed?
A. Volcanic eruption
B. Corals growing around a volcanic island
C. Underwater bedrock formations
D. Earthquake
44. What is a long shore drift?
A. Movement of sand and shingles along the coast
B. Sand bars
C. Accumulation of sad at the river mouth
D. Island formed by volcanic eruptions
45. How does an occlusion form?
A. Cold air moving up from the ground
B. Cold front pushing warm air up of the ground
C. Unbalance electrical reaction in the air
D. Cold and warm air mixing in the atmosphere
46. What is a heat haze?
A. A reflection caused by pollutants in the air
B. A distorted image resulting from the bending o sun’s light rays by changes in air temperature
C. A movement o warm air over a vast expanse of land
D. Caused by extremely high temperature common in dessert areas
47. What sort of rock formation do the world’s greatest mountain ranges consist of?
A. Magma C. Fold eruptions
B. Chalk deposit D. Slip formation
48. What is the force that wears down mountains?
A. Earthquake C. Volcanic eruptions
B. Erosion D. Deforestation
49. How are volcanic islands formed?
A. Collision of two oceanic plates
B. Cooling of lava by seawater
C. Volcanic eruptions
D. Accumulation of corals
50. Ang may-akda ng “Kahapon ,Ngayon at Bukas” ay si ________.
A. Aurelio Tolentino B. Juan Abad C. Alejandro Abadilla D. Severino Reyes
51. Isang awiting bayan na tungkol sa paglilibing ang _______.
A. umbay B. kundiman C. sambotani D.soliranin
52. The highest mountain in the continent of Africa is ______ .
A. Mt.Everest B. Mt.Kilimanjaro C. Mt. Fuji D. Mt. Apo
53. The “yellow race “ is also known as :
A. Caucasians B. Mongoloids C.Negroids D.Malays
54. The function by which schools help children to participate effectively in larger society is called :
A. Socialization B. Education C. Enculturation D. Acculturation
55. What phase of the Moon is observed when it cannot be seen at all because it passes directly
between the earth and the Sun ?
A. New Moon B. First quarter C. Full Moon D. Last quarter
56. May sagisag panulat na Paralitiko at ang tinaguriang “Utak ng Himagsikan” , ay si _______.
A. Emilio Jacinto B. Antonio Luna C. Jose Corazon de Jesus D. Apolinario Mabini
Below is a Biographical sketch of an American movies writer:
How Did Elvis Presley Achieve Recognition
Success often comes to those with humble beginnings, Elvis Aaron Presley was born on January
8, 1935, in Tupelo, Mississippi. He first sang in a church choir and taught himself to play the guitar, but
he never learned to read music. By 1953, he had moved to Memphis, Tennessee, graduated from high
school, and enrolled in night school to become an electrician. That year, at Sun Records, Presley recorded
a personal record for his mother, a song that was heard by the company's president. As a result of the
president's recognition, Presley's first record, "That's all Right, Mama," was out in 1954.
He toured the South, and in 1955 five of his records were released simultaneously. His first
national television appearance was that year on Jackie Gleason's "The Stage Show," but Presley became
known for his appearance on "The Ed Sullivan Show," where the young singer gyrated as he sang "rock n'
roll" music. During the live television performance, Presley was photographed only from the waist up
because his motions were considered obscene.
"Elvis the Pelvis" began his film career in 1956 with LOVE ME TENDER and signed a long term
film contract. The movie critics were not always kind, but teenagers flocked to Presley's films. Within a
few short years, Presley had established a career that would span twenty-five years of ups and downs
and make him one of the most popular entertainers in history. Long after his untimely death at age 42,
Presley would be remembered as "The King of Rock n' Roll”
57. The statement that "success often comes to those with humble beginnings" would apply best to
which of the following figures?
A. Ramon Magsaysay
B. Gloria M. Arroyo
C. Corazon C. Aquino
D. Joseph E. Estrada
58. The main idea of the sketch is that
A. singers are more successful if they appear in films
B. there has always been obscenity on television
C. opportunity and luck are often as important as hard work
D. celebrities are usually more famous after their death
59. The last sentence reveals that the author's attitude toward Presley is one of
A. indifference
B. admiration
C. disbelief
D. disgust
The poem below is entitled "Suburban Prophecy" which is written by Howard Nemerov.
On Saturday, the power-mowers' whine
Begins the morning. Over this neighbourhood
Rises the keening, petulant voice, begin
Green oily teeth to chatter and munch the cud.
Monsters, crawling the carpets of the world,
Still send from underground against your blades
The roots of things battalions green and curled
And tender, that will match your blades with blades
Till the revolted throats shall strangle on .
The tickle of their dead, till straws shall break
Crankshafts like camels, and the sun go down
On dinosaurs in swamps. A night attack
Follows and by the time the Sabbath dawns
All armored beasts are eaten by their lawns.
60. How long does the action of the poem take place?
A. A week
B. Twenty-four horns
C. An afternoon
D. A morning
61. The poet's use of words such as whine, voice, teeth, chatter and munch is to suggest that the
power-mowers are
A. very powerful
B. alive
C. like cows
D. green
62. To what does the phrase "your blades" in line 8 refer?
A. Lawnmowers
B. Roots
C. Carpets
D. Monsters
63. The imagery in the first stanza appeals to the reader's sense of
A. sight
B. touch
C. smell
D. hearing
Ang sumusunod ay isang talumpati na may pamagat na SA KABATAAN na sinulat ni Onofre Pagsanghan.
Isa sa mga salitang napag-aralan natin sa wikang Pilipino ay ang salitang NABANSOT.
Kapang ang isang bagay daw ay dapat pang lumaki ngunit ito'y tumigil na sa paglaki, ang bagay na ito
raw ay NABANSOT. Marami raw uri ng pagkabansot ngunit ay pinakamalungkot na uri raw ay ang
pagkabansot ng isipan, ng puso at ng diwa.
Ang panahon ng kabataan ay panahon ng paglaki, ngunit ang ating paglaki ay kailangang paglaki,
at pag-unlad ng ating buong katauhan, hindi lamang ng ating sukat at timbang. Kung ga-poste man ang
ating taas at ga-pison man ang ating bigat ngunit kung ang pag-iisip naman nati'y ga-kulisap lamang kay
pangit na kabansutan. Kung tumangkad man tayong tangkad-kawayan at bumilog man tayong bilog-
tapayan, ngunit kung tayo nama'y tulad ni "bondying" ay di mapagkatiwalaan-anong laking kakulangan.
Kung magkakatawan tayong katawang "Tarzan" at mapatalas ang ating isipang sintalas ng kay Rizal,
ngunit kung ang ating kalooban nama'y itim na duwende ng kasamaan-anong kapinsalaan para sa
kinabukasan.
Kinabukasan, kabataan, tayo raw ang pag-asa ng Inang Bayan. Tayo raw ang maghahatid sa
kanya sa langit ng kasaganaan at karangalan o hihils sa kanya sa putik ng kahirapan at kahihiyan. Ang
panahon ng pagkilos ay ngayon, hindi bukas, hindi sa isang taon. Araw-araw ay tumutuwid tayong
palangit or bumabaluktod tayong paputik. Tamang-tama ang sabi ng ating mga ninunong kung ano raw
ang kinamihasnan ay siyang pagkakatandaan. Huwag nating akalaing makapagpapabaya tayo ng ating
pag-aaral ngayon at sa araw ng bukas ay bigla tayong magiging mga dalubhasang magpapaunlad sa
bayan. Huwag nating akalaing makapaglulublob tayo ngayon sa kalaswaaan at kahalayan at sa
mahiwagang araw ng bukas sigla tayong magiging ulirang mga magulang.
Kabataan, tunay na pag-ibig sa bayan, ang tunay na nasyonalismo, ay wala sa tamis ng pangarap
wala rin sa pagpag ng dila. Ang tunay na pag-ibig ay nasa pawis ng gawa.
64.Sa alin makikita ang tunay na NASYONALlSMO?
A. Diwa
B. Gawa
C. Sulat
D. Salita
65. Alin salita ang paulit-ulit na binabanggit ni Onofre Pagsanghan?
A. Nabansot
B. Bayan
C. Kabataan
D. Kung
66. Bakit di dapat tumangkad tulad ni "bondying"?
A. Di ito magpakakatiwalaan
B. Di totoo ito
C. May kakulangan ito
D. Magulo kasi ito
67. Alin sa mga sumusunod ang mensahe ng talumpati?
A. Ang mataas na paniniwala at taimtim na pananalig ay kailangang taglayin upang ang hangarin
sa buhay ay ating kamtin.
B. Ang panahon ng kabataan ay panahon ng paglaki at pagbabagong makabuluhan.
C. Ang gawa ang siyang sukat ng kadakilaan.
D. Ang kabataan ay siyang pag-asa ng bayan.
68. What is meant by TWO PINS in this sentence?
For two pins I could have hit him on the nose.
A. A second course of action
B. Without much persuading
C. The second chance
D. Have a second alternative choice
69. What is meant by AT SIXES AND SEVENS in this sentence?
We moved into the house last week, but I'm afraid everything, is still at sixes and sevens.
A. The things have not been shipped.
B. In a state of confusion
C. In an orderly manner
D. The boxes are still intact
70. Which word ends with [S] pronounced [Z]?
A. Maps
B. Jakes
C. Laughs
D. Buys
71. What is meant by SOFT OPTION in these sentences?
Rebecca realized that if she stayed in her present job it would mean competing with an envious rival.
Leaving the company would probably be a soft option.
A. An action that is difficult to take
B. An action that is not agreeable
C. An action that is easier.
D. An action that is weakly funded
72. Which word contains the voiced Th?
A. Thank
B. These
C. Think
D. Thing
73. To gain the attention of the audience, the trick is __________?
A. start low, speak hurriedly
B. start high, speak loudly
C. start high, speak rapidly
D. start low, speak slowly
74. Which word contains the [ae] sound?
A. Carriage
B. Cabin
C. Caste
D. Can
75. Which of the following words DOES NOT contain the [voiceless th]?
A. Mouth
B. Breath
C. Teeth
D. Health
76. Which pitch is used for the word STRANGE?
What a strange story!
A. 3
B. 2
C. 4
D. 1
77. What do the following lines CONVEY?
Midnight, not a sound from the pavement.
Has the moon lost her memory?
She is smiling alone.
In the lamp light the withered leaves
Collect at my feet
And the wind begins to moan
A. Confusion
B. Optimism
C. Loneliness
D. Eagerness
78. I can’t find ________ calculator; may I use _________, Prince?
A. My, yours
B. His, yours
C. My, his
D. Yours, mine
79. Neither the teacher nor the students ________________ present in the affairs.
A. Am
B. Was
C. Is
D. Were
80. We admire ___________ when Cris Paner speaks English fluently with _______________.
A. Him, them
B. Her, us
C. Him, we
D. Him, me
81. “The figures must be TRANSMUTED in order to understand the grade.” The capitalized word means:
A. Estimated
B. Surpassed
C. Changed
D. Summed
82. Some preachers suggest the _____________ that the end of the world is near.
A. Proposal
B. Prophecy
C. Prophetic
D. Prophet
83. Filipino writer whose stories and poems depict Filipino-Spanish cultural beliefs and traditions.
A. Edilberto Tiempo
B. N.V.M. Gonzales
C. Bienvenido Santos
D. Nick Joaquin
84.The people of Montalban in Rizal raised protest on the landfill since.
A.Water seepage would pollute water source
B.People were disturbed by garbage trucks
C.Air pollution could not be controlled
D.Garbage cold be recycled
85.The weather report recorded a temperature of 29°C in the morning and 33.5°C at two O’clock in the
afternoon. How many degrees higher was the afternoon temperature
A. 4.5°C B. 5.8°C C. 6.3°C D. 5.5°C
86.Why are Filipino overseas workers considered modern time heroes?
A.They sacrifice all-the family life and economies just to improve their quality of life.
B.They became broad minded, skilled and economically stable.
C.They have helped stabilize Philippine economy
D.They come home ready to feast their townmales and friends
87.Hindi naman pangit ang larawan ngunit _____ang kanyang anyo.
A. pinapuri – puri C. kapuri-purihan
B. kapuri – puri D. kapurihan
88. I have one life to give and I give it al to my country.” This pictures a:
A. a soldier at war
B. worker in a cement factory
C. a policeman on the street
D. a doctor in the operating room.
89. The endangered species are protected by DENR. All these are concerns of:
A. Tourism C. Urbanization
B. Industrialization D. Terrorism
90. Abuses to soldiers, women, children and the helpless are attended to by a Commission. Who was the
first Commissioner on Human Rights?
A. Hon. Aurora Recine
B. Hon. Mary Conception Bautista
C. Hon. Teofisto Guingona
D. Hon. Sedfrey Ordonez
91. After a series of tutorial lessons at PANER Tutorial Center, Sophia’s grade in Math rose from 78 to 88.
By what percent did her average increase?
A. 35.75% C. 11.36%
B. 12.82% D. 20.16%
92. If four mangoes cost ₱29.00, at that price, what will 2 ½ dozen mangoes cost?
A. ₱370.00 C. ₱188.50
B. ₱348.50 D. ₱217.50
93. A room of 10m by 7m. There is a 7.5m by 5m carpet in the middle. What percent of the room is
uncovered?
A. 46.4% C. 48%
B. 53.6% D. 80%
94.One angle of a parallelogram is 35°. What are the measures of the three other angles?
A. 145°, 35°, 145° C. 85°, 135°, 140°
B. 45°, 65°, 170° D. 35°, 65°, 65°
95.If the scores of 10 students are 76, 80, 75, 83, 80, 79, 85, 80, 88, 90, the mode is___
A. 79 C. 80
B. 85 D. 88
96. Which of the following methods can all diabetics control their condition and avoid heart disease and
blindness?
I. Regulating their intake of glucose
II. Increasing the levels of insulin in the body by taking insulin injections
III. Maintaining a reasonable exercise regimen to keep weight down.
A. I B. II C. I and II D. I and III
97. The principle under which a thermostat operates is the same when
A. a gas expands to fill the container in which it is held.
B. a pendulum swings when it is set into motion.
C. a chemical reaction occurs when two substances combine.
D. the level of mercury rises or falls in a glass tube.
98. Why does a bullet when discharged into the air eventually fall to the ground? This is due to the Law of
A. Universal Gravitation B. Applied Force C. Inertia D. Action and Reaction
99.The stanza below is taken from "Barter" by Sara Teasdale.
Life has loneliness to sell,
Music like a curve of gold,
Scent of pine trees in the rain,
Eyes that love you, arms that hold,
And for your spirit's still delight,
Holy thoughts that star the night.
To what does Teasdale compare music?
A. The scent of pine trees B. A curve of gold C. Eyes that love D. The rain
100.An elderly woman suffered a stroke, a restriction of blood flow to the brain. If the stroke caused to
the right side of her body to become temporarily paralyzed, she most likely experienced a decreased blood
flow to
A. the left side of her body. B. the front of her brain.
C. the left side of her brain. D. the right side of her brain.
101.Which is the BEST WAY to write the underlined portion of this sentence?
Many viewers taped shows to watch later.
A. tapped B. had taped C. tape D. had tapped
102.Alin ang kahulugan ng KAHIRAMANG SUKLAY?
A. Kakilala B. Kaibigan C. Karibal D. Kalahi
103.Ano ang pinakaangkop na kahulugan nito?
Nagsasaya tayo ngayon sapagkat ang iyong namatay na kapatid ay muling nabuhay; ang nawawala ay
muling nakita.
A. Ang pagbabalik ay dapat ipaghanda nang malaki.
B. Ang pagbabago ng kapatid ay dapat pahalagahan.
C. Dapat silang magsaya sa muli nilang pagsasama-sama
D. Ang pagsasama nila ay dahil sa muling pagbabalik ng kapatid.
104.Two days after Japan attacked Pearl Harbor, Roosevelt made the following statements:
In the past few years and most violently in the past few days, we have learned a terrible lesson. We must
begin the great task that is before us by abandoning once and for all the illusion that we can never again
isolate ourselves from the rest of humanity.
In the statement, Roosevelt is expressing the ideas of __________.
A. an internationalist B. an anti-imperialist C. an imperialist D. an isolationist
105.Which pitch is used for the word STRANGE?
What a strange story!
A. 3 B. 2 C. 4 D. 1
106.What is the function of DIFFUSION in the human body?
A. Regulates blood flow
B. Plays an insignificant role in the body's functioning
C. Allows an even distribution of substances throughout all cells of the body
D. Comes into play in times of extreme illness
107.Utang sa kanyang sipag at sikap sa paggawa ang kanilang maalwang pamumuhay.
A. Pangngalan B. Pangngalang-diwa C. Pawatas D. Pang-ukol
108.Which of the following represent, ethnocentric behavior?
A. A tourist who lectures his foreign hosts on the "uncivilized" nature of their marriage customs
B. A student who tutors an immigrant in English
C. A Hispanic community group demands that public aid forms be published in English
and Spanish.
D. A Peace Corps volunteer who helps dig wells in Central Africa
109.The following are legitimate children, EXCEPT
A. those born by artificial insemination.
B. those legitimated.
C. those born during a valid marriage of parents.
D. those born out of a valid marriage of the parents.
Write the meaning of the underlined idiom from the choices.
110. There's too much red ink in the company's financial statement.
A. Financial loss C. Financial debt
B. Financial gain D. Financial obligation
111. In doing things, you must always bear in mind this ''Rome was not built in a day"
A. Accept and bear consequences of your own doing
B. Hatred between forgiven two parties.
C. Great things require time & effort
D. Always on the move
112. Amiel is paddling on his own canoe because he wants to concentrate on his thesis.
A. To work overtime C. To work well in groups
B. To work without help D. To work with the help of someone else
113. Jona is a responsible sister. She always takes care of here siblings. In the sentence, she refers to
Jona, so it is called the:
A. Antecedent C. Predicate noun
B. Reference D. Pronoun
114. _____ the floods that damaged their crops, the farmers are still hopeful of an abundant harvest.
A. Spite of C. In spite
B. Despite D. Despite of
115. Most teachers are familiar _____ the communicative language teaching approach.
A. By C. With
B. In D. Of
116. The teacher not the academic managers _____ to attend the workshop on Outcomes Based
Education two days from now.
A. Are going C. is going
B. Were going D. Was going
117. All students must submit his homework tomorrow.
A. Student C. His
B. their D. Tomorrow
118. Let's go to the nearest store, isn’t it ?
A. Let's go C. The
B. Store D. Will we
119. ''A rabbit ran across the street with long ears". Which sentence error is exhibited in the sentence?
A. Misplaced modifier C. Dangling modifier
B. Run-on sentence D. Faulty diction
120. If I _____ the money, I would buy a yacht.
A. Had C. Have
B. Had had D. Have had
121. A. Then you will come to a hallway leading to the library's music room
B. Walking around the information desk to the left, you will pass children's reading room
C. That leads to the music room
D. You enter the main door of the library, you see the information desk directly in front of you
E. At the end of the hallway you will see a sign
What is the proper sequence of the sentence?
A. DBAEC C. BDAEC
B. BDEAC D. DBEAC
122.Siya ay si “Dolores Manapat”.
A. Graciano Lopez Jaena B. N.V.M. Gonzalez C. Andres Bonifacio D. Marcelo H. del Pilar
123. The process of comparing one quantity with corresponding standard is called ______ .
A. Measurement B. Formulation C. Interpolation D. Extrapolation
124.Matter is anything that has ______.
A. Weight and volume B. Mass and volume C. Weight and mass D. Mass and shape
125. A student walks from home to school 3 blocks away and then returns home for lunch. After lunch,
he goes back to school. His total displacement is _______ .
A. 0 block B. 3 block C. 6 blocks D. 9 blocks
126. What property of matter is involved in the sign, “Fasten Your Seat Belt”?
A. Impenetrability B. Inertia C. Acceleration D. Interaction
127. Which of the following situations is true as a boat moves from salty sea to fresh water river?
A. It sinks deeper. B. It floats higher. C. It stays at the same level. D. It moves faster.
128. Which of the following is conserved when a pendulum is swinging?
A. Mass B. Momentum C. Energy D. Weight
129. Which statement best explains why ice is a good refrigerant ?
A. It cools by evaporation. B.It absorbs heat while melting.
C.It conducts heat quickly. D.It does not melt inside the refrigerator.
130. An object becomes positively charge if it _________ .
A. gains protons B. gains electron C. loses protons D. loses electrons
131. The force that holds the atoms and molecules together to form larger bodies is known as
A. Gravitational force B. Electromagnetic force
C. Weak nuclear force D. Strong nuclear force
132. What is the maximum number of electrons that can be accommodated in n=4?
A. 2 B. 8 C. 16 D. 32
133. It is characterized by shared experiences and mutual responsibilities.
A.Gesellschaft B. Gemeinchaft C. Enculturation D. Acculturation
134. It features the exchange of goods, money and services .
A. Gesellschaft B. Gemeinchaft C. Enculturation D. Acculturation
135. The longest revolt in the Philippine History is called the ______ .
A. Dagohoy revolt B.Diego Silang revolt C. Gabriela Silang revolt D. Palaris revolt
136. Violation of human rights is a violation of the _____ of persons.
A. dignity B. intelligence C. emotions D. freedom
137. Which of the following chronological order for the events in history listed below?
I. Execution of Dr. Jose Rizal
II. Declaration of the First Philippine Republic
III. Edsa Revolution
IV. World War II
A. I, IV, II, III C. II, I, III, IV
B. II, I, IV, III D. I, II, IV, III
138. Which primary group is considered the "nursery of human nature"?
A. Play group C. Classmate's group
B. Peer group D. Family group
139. One of the strengths of the Filipino character is "pakikipagkapwa-tao". This is manifested in one of
the following except one:
A. Pakikiramdam C. Pakikiramay
B. Malasakit D. Lakas ng loob
140. In every society, certain position are assigned to individuals on the basis of age, sex, marital status,
and similar to other criteria. These patterns are:
A. University accepted and practice C. Taught in schools
B. Learned from parents' D. Culturally defined
141. Which of the following action must be done to show perseverance if you failed in your project the
first time you do it?
A. Stop doing the project and try a much easier one
B. Tell tour teacher you will not do it anymore
C. Try and try until you produce one
D. Ask somebody to do it for you
142. Which of the following scientific attitude is shown by the person who keeps seeking answer to
things he is interested with?
A. Curiosity C. Open mindedness
B. Honesty D. Resourcefulness
143. Democracy refers to the involvement of all citizens in the control of government, while an oligarchy
relegates control of the government to:
A. An autocrat C. The mother country
B. A select few D. Religious group
144. The benevolent assimilation proclamation on December 21, 1898, was the first official indication of
American policy regarding the Philippines. It is expressly indicated the intention of the United States to
stay in the Philippines by exercising the right of sovereignty over the Filipinos. Who was the President
who issued this "Benevolent Assimilation" policy?
A. Eisenhower C. McKinley
B. Wilson D. Roosevelt
145. What is the social scientist's explanation of the relationship of a leader to history?
A. Leaders are a product of their times
B. History is a reflection of great leaders
C. Every great movement is the lengthened shadow of a great man
D. Had any great leader been different than he was, the course of would have been different
146. Which of the following forces of social change have the greatest impact upon the traditional
Filipino family in the 20th century?
A. Immigration and Migration
B. Socialization and Stratification
C. Specialization and Assimilation
D. Industrialization and Urbanization
147. Through the process of socialization, the individual is able to:
A. Function as contributing member of the group
B. Learn to dialogue with peers and adult
C. Satisfy his personal needs and drives
D. Become an adult member of society
148. Emilio Jancinto's literary works were written in Tagalog and small part in Spanish. The best in all of
his works was written o October 8, 1897. it was visibly an animation of Rizal's ''Mi Ultimo Adios”.
What was the title of his poem?
A. Mi Madre C. Ala Juventud Filipina
B. Ala patria D. Amor Patrio
149. The committe recommends that she _____ the proposal on time.
A. Submit C. Submitted
B. Submits D. Will submit
150. If I were you, I _____ not be joining the contestant.
A. Will C. Would
B. Were D. Will be
151. We _____adhere to the school's policy that ''no down payment, no enrollment.
A. Strict C. Strictly
B. Strictness D. Strictest
152. It's a necessity, I _____ see my thesis adviser.
A. Could C. May
B. Shall D. Need to
153. if you had saved your money, you _____ your nephew to school.
A. Would send C. would have send
B. Send D. will send
154. The school director suggests that the suspended student _____ a written explanation.
A. Makes C. Made
B. Make D. Will make
155. I want to find out which is better _____ Superman, X-men and Spider man.
A. Between C. Among
B. From D. With
156. They _____ to Baguio for five hours when the hard rain fell.
A. Had driven C. Have been driving
B. Have driven D. Had been driving
157. She is fond _____ stamps and stationeries.
A. By collecting C. For collecting
B. In collecting D. Of collecting
158. The rich _____ the most powerful in the society
A. Was C. Is
B. Were D. Are
159. I think one or two of the guests' _____ here now.
A. Are C.Was
B. Were D. Is
160. Speaker 1: Where were you last night?
Speaker 2: Last night? I am at Starbucks SM Manila.
The speaker who committed an error is:
A. Speaker 1 C. Speaker 2
B. Both speakers D. None of the above
161. The Nibelungenlied is a
A. Latin Myth
B. Chinese legend
C. Russian folk song
D. Medieval German epic
162. An association wherein the name of something is substituted by something that represents it.
A. Metonymy
B. Comparison
C. Euphemism
D. Personification
163. Because the moon rotates on its axis at the same time as it ______ around the earth, we see the
same side
A. Revolve
B. Revolves
C. Is revolving
D. Has been revolving
164. In English verse, a poetic foot having 1 stressed syllable followed by 1 unstressed syllable is ______.
A. Trochaic
B. Iambic
C. Dactylic
D. Anapaestic
165. Senators were accused by activists of washing their hands with the perfumes of Arabia. This state is
commonly known as
A. Guilt
B. Triumph
C. Indecision
D. Aggression
166. It is a collection of religious poetry written by Rabindranath Tagore.
A. Mahabharata
B. Gitanjali
C. The Ramayana
D. Bhagavad Gita
167. I suggest that he _____ in the room for one week.
A. Stay
B. Stayed
C. Staying
D. Stays
168. The manager told his workers, “We have to reduce our workforce.” What did he mean?
A.Workers are free to leave
B.Workers are warned of possible lay off
C.Workers have to double time on their jobs
D.Workers should submit themselves to a reducing gym
169. The parent remarked, “__________ I come late, just lock the door.”
A. In the absence
B. In the process
C. In the event
D. In the case
170. The copyreader found the news story boring. He found it full of _____.
A. Adjectives
B. Verbs
C. Pronouns
D. Adverbs
171. There were three guests on the stage. They were made up of a parent, the governor and the
principal. Who should be acknowledged first by the valedictorian?
A. The classmates
B. The principal
C. The governor
D. The parent
172. What is suggested in the opening line? June 13, 1986 - they came from all over America - 200,000
heroes strong, with their families.
A. The writer holds great admiration for the veterans
B. The writer is opposed to the Vietnam War
C. The writer was a veteran of the war
D. The writer is a flag-waving patriot
173. A readability mismatch happens when the reading levels of books exceed the reading levels of the
students. In this situation, the students experience frustration and they fall short of the expected or
desired output. A student who finds himself/ herself in such a mismatch will likely do which of the
following?
A. Give an intelligent critique of the selection or story read
B. Write a comprehensive reaction paper regarding the selection or story read
C. Present an argument that the selection or story read was not properly written
D. Manifest an expected and commensurate emotional reaction to the selection or story read
174. What does this mean: Excuses are for losers… those who take responsibility for their actions are
real winners in life. This tells of?
A. losers often fail because they find reasons for losing
B. a winner can also be a quitter
C. accountability of one’s action tells of bravery
D. excuses are needed to justify any failure
175. A couple accepted a wedding invitation. They showed pleasure in these remarks
A. All guests congratulated the organizers and the couple
B. Guests came in and out of the ceremonies
C. The ceremonies were very impressive
D. The couple felt uneasy with the priest
176. What correction should be made to this sentence?
One of the theories is that the first child receives more of the parents' attention than other children so
first-borns tend to be more intellectual.
A. Change is to are
B. Insert a comma after children
C. Change parents' to parent's
D. Change theories to theory's
177. Carl Sundburg wrote "Jazz Fantasia" which has for its first stanza:
Drum on your drums, batter on your banjos, sob on the long cool winding saxophones. Go to it, O
jazzmen.
Which words illustrate alliteration?
A. Batter and banjos
B. Sob and winding
C. Long and cool
D. To and it
178. What is meant by AT SIXES AND SEVENS in this sentence?
We moved into the house last week, but I'm afraid everything, is still at sixes and sevens.
A. The things have not been shipped.
B. In a state of confusion
C. In an orderly manner
D. The boxes are still intact
179. In the opening of the free trade, if world prices of a good are less than domestic prices of that same
good, _________.
A.domestic consumers will experience a loss of surplus.
B.domestic prices will drop to the world price level.
C.all domestic producers of that good will try to find another market because they can’t compete with
foreign producers.
D.domestic producers will increase the quantity supplied in order to crowd out the foreign produced
goods.
180. It states that as the price of the commodities increase the amount of goods the consumer is willing
to purchase decrease and as the price of the commodities decrease the willingness of the consumer to
buy increases and other factor remain constant.
A.Law of Diminishing Marginal Utility
B.Law of Gravity
C.Law of Supply
D.Law of Demand
181. A deliberate attempt to recognize and transform existing agrarian system with the intention of
improving the distribution of agricultural incomes and thus fostering rural development.
A.Millennium Development Plan C.Water Reform
B.Land Reform D.Development Goals
182. What is the process by which the productive capacity of the economy is increased over time to
bring about rising levels of national output and income?
A.Economic growth C.Economic development
B.Industry D.Employment
183. A system whereby the determination of exchange rate is left solely to the market forces.
A.Foreign exchange liberalization
B.Import liberalization
C.Terms of trade
D.Foreign investment
184. All are possible results when a high population growth rate continues in the Third World except
A.growth of slums
B.spread of diseases due to poverty and poor sanitation
C.not enough schools, hospitals, roads, bridges, etc.
D.increased Gross National Product
185. Which of the following is the nature of power of taxation?
A.It is inherent in sovereignty.
B.It is legislative in nature.
C.It is subject to constitutional and inherent limitations.
D.All of the above
186. A kind of tax based on the rate of which decreases as the tax base or bracket increases.
A.Progressive C. Regressive
B.Graduated D. Proportional
187. Agrarian reform program, Philippine experience is a success.
A.The statement is generally true.
B.The statement is doubtful.
C.The statement is untrue.
D.There is no basis to conclude.
188. It is also known as the Comprehensive Agrarian Reform Law (CARL)
A.Presidential Decree # 2
B.Presidential Decree # 27
C.Republic Act 6657
D.Republic Act 5766
189. The Cooperatives Development Program of the government is designed primarily to support the
agrarian reform program. It aims to achieve a dignified existence for the small farmers free from
pernicious institutional restraints and practices.
A.Only the first statement is true and correct.
B.Only the second statement is true and correct.
C.Both statements are true and correct.
D.Both statements are untrue and incorrect.
190. The words “inappropriate, illegal, irresponsible and unaware” have prefixes which are classified
as__________.
A. positive C. negative
B. common D. neutral
191. Using contextual clues, complete the following statement. Most flowers________ in spring.
A. bloom C. wither
B. weaken D. die
192. Which does NOT belong?
A. Foot C. Centigrade
B. Kilometer D. Mile
193. Which passage is expressed effectively and without structural error/s?
A. Having called the other students and they, the secretary went home.
B. Having called the other students and ourselves the secretary went home.
C. Having called the other students and we, the secretary went home.
D. Having called the other students and us, the secretary went home
194. When I met Albert yesterday, it was the first time I_____ her since Christmas.
A. have seen C. have been seen
B. saw D. had seen
195. The measure of choosing well is whether or not a man likes what he__
A. has chosen C. chose
B. is choosing D. choose
196. Your father is paying for your plane ticket, _?
A. isn’t it C. isn’t he
B. is he D. aren’t you
197. Alin ang angkang pinagmulan ng mga wika sa Pilipinas?
A. Indones B. Malay
C. Indones Polinesyo D. Malayo Polinesyo
198. Alin sa mga sumusunod ang salitang pambansa?
A. Pinoy C. Mapagkumbaba
B. Kamusta D. Nagdadalantao
199. Ang paagtuturo ng Filipino bilang pangalawang wika ay higit na magiging mabisa kung gagami
A. pagsasaulo B. tanong-sagot
C. pagkukuwento D. modelo
200. Ang pagkautal ay matatawag na sagabal sa pagsasalita.
A. saykolohikal C. pisikal
B. pisyolohikal D. semantiko

More Related Content

What's hot

Professional education set d (with highlighted answers)
Professional education set d (with highlighted answers)Professional education set d (with highlighted answers)
Professional education set d (with highlighted answers)Lucille Clavero
 
Professional education set a (with highlighted answers)
Professional education set a (with highlighted answers)Professional education set a (with highlighted answers)
Professional education set a (with highlighted answers)Lucille Clavero
 
Professional education set e (without highlighted answers)
Professional education set e (without highlighted answers)Professional education set e (without highlighted answers)
Professional education set e (without highlighted answers)Lucille Clavero
 
Let professional education 9
Let professional education 9Let professional education 9
Let professional education 9Alex Acayen
 
Let professional education 3
Let professional education 3Let professional education 3
Let professional education 3Alex Acayen
 
Final coaching questions in prof. education by prof. cris paner
Final coaching questions in prof. education by prof. cris panerFinal coaching questions in prof. education by prof. cris paner
Final coaching questions in prof. education by prof. cris panerArneyo
 
Licensure examination for teachers set 2 part 1
Licensure examination for teachers set 2 part 1Licensure examination for teachers set 2 part 1
Licensure examination for teachers set 2 part 1Arneyo
 
Let professional education 1
Let professional education 1Let professional education 1
Let professional education 1Alex Acayen
 
Professional education set b (with highlighted answers)
Professional education set b (with highlighted answers)Professional education set b (with highlighted answers)
Professional education set b (with highlighted answers)Lucille Clavero
 
Professional education set d (without highlighted answers)
Professional education set d (without highlighted answers)Professional education set d (without highlighted answers)
Professional education set d (without highlighted answers)Lucille Clavero
 
Licensure examination for teachers
Licensure examination for teachersLicensure examination for teachers
Licensure examination for teachersMina Olario
 
Let prof ed reviewer
Let prof ed reviewerLet prof ed reviewer
Let prof ed reviewerbmsg wap
 
Sample preboard-exam-prof ed
Sample preboard-exam-prof edSample preboard-exam-prof ed
Sample preboard-exam-prof edJohn Paul Intano
 
Let professional education 7
Let professional education 7Let professional education 7
Let professional education 7Alex Acayen
 
Licensure examination for teachers set 1 part 1
Licensure examination for teachers set 1 part 1Licensure examination for teachers set 1 part 1
Licensure examination for teachers set 1 part 1Arneyo
 
Licensure examination for teachers1
Licensure examination for teachers1Licensure examination for teachers1
Licensure examination for teachers1Mina Olario
 
Professionaleducationseta
ProfessionaleducationsetaProfessionaleducationseta
ProfessionaleducationsetaArneyo
 
Reviewer LET professional education 12
Reviewer LET professional education 12Reviewer LET professional education 12
Reviewer LET professional education 12Daniel Bragais
 

What's hot (20)

Professional education set d (with highlighted answers)
Professional education set d (with highlighted answers)Professional education set d (with highlighted answers)
Professional education set d (with highlighted answers)
 
Professional education set a (with highlighted answers)
Professional education set a (with highlighted answers)Professional education set a (with highlighted answers)
Professional education set a (with highlighted answers)
 
Akon3
Akon3Akon3
Akon3
 
Professional education set e (without highlighted answers)
Professional education set e (without highlighted answers)Professional education set e (without highlighted answers)
Professional education set e (without highlighted answers)
 
Let professional education 9
Let professional education 9Let professional education 9
Let professional education 9
 
Let professional education 3
Let professional education 3Let professional education 3
Let professional education 3
 
Final coaching questions in prof. education by prof. cris paner
Final coaching questions in prof. education by prof. cris panerFinal coaching questions in prof. education by prof. cris paner
Final coaching questions in prof. education by prof. cris paner
 
Licensure examination for teachers set 2 part 1
Licensure examination for teachers set 2 part 1Licensure examination for teachers set 2 part 1
Licensure examination for teachers set 2 part 1
 
Let professional education 1
Let professional education 1Let professional education 1
Let professional education 1
 
Professional education set b (with highlighted answers)
Professional education set b (with highlighted answers)Professional education set b (with highlighted answers)
Professional education set b (with highlighted answers)
 
Professional education set d (without highlighted answers)
Professional education set d (without highlighted answers)Professional education set d (without highlighted answers)
Professional education set d (without highlighted answers)
 
Licensure examination for teachers
Licensure examination for teachersLicensure examination for teachers
Licensure examination for teachers
 
Akon1
Akon1Akon1
Akon1
 
Let prof ed reviewer
Let prof ed reviewerLet prof ed reviewer
Let prof ed reviewer
 
Sample preboard-exam-prof ed
Sample preboard-exam-prof edSample preboard-exam-prof ed
Sample preboard-exam-prof ed
 
Let professional education 7
Let professional education 7Let professional education 7
Let professional education 7
 
Licensure examination for teachers set 1 part 1
Licensure examination for teachers set 1 part 1Licensure examination for teachers set 1 part 1
Licensure examination for teachers set 1 part 1
 
Licensure examination for teachers1
Licensure examination for teachers1Licensure examination for teachers1
Licensure examination for teachers1
 
Professionaleducationseta
ProfessionaleducationsetaProfessionaleducationseta
Professionaleducationseta
 
Reviewer LET professional education 12
Reviewer LET professional education 12Reviewer LET professional education 12
Reviewer LET professional education 12
 

Similar to Final coaching questions in general education by prof. cris paner

General education - 1.docx
General education - 1.docxGeneral education - 1.docx
General education - 1.docxRoyCabarles3
 
General education - 1.docx
General education - 1.docxGeneral education - 1.docx
General education - 1.docxNoraima2
 
Gen-Ed-September-2016.docx
Gen-Ed-September-2016.docxGen-Ed-September-2016.docx
Gen-Ed-September-2016.docxLalaGonzal
 
General education set a with highlighted answers)
General education set a with highlighted answers)General education set a with highlighted answers)
General education set a with highlighted answers)Lucille Clavero
 
UNIFIED PRE-TEST IN INTRODUCTION TO THE PHILOSOPHY PPT.pptx
UNIFIED PRE-TEST IN INTRODUCTION TO THE PHILOSOPHY PPT.pptxUNIFIED PRE-TEST IN INTRODUCTION TO THE PHILOSOPHY PPT.pptx
UNIFIED PRE-TEST IN INTRODUCTION TO THE PHILOSOPHY PPT.pptxJenniferRayaTalaugon
 
Let general education 10
Let general education 10Let general education 10
Let general education 10Alex Acayen
 
Let general education 2
Let general education 2Let general education 2
Let general education 2Alex Acayen
 
Let general education 7
Let general education 7Let general education 7
Let general education 7Alex Acayen
 
Let professional education 10
Let professional education 10Let professional education 10
Let professional education 10Alex Acayen
 
2016 20 let_20bse_20pre-board_20with_20answer
2016 20 let_20bse_20pre-board_20with_20answer2016 20 let_20bse_20pre-board_20with_20answer
2016 20 let_20bse_20pre-board_20with_20answerArneyo
 
Let professional education 4.2
Let professional education 4.2Let professional education 4.2
Let professional education 4.2Alex Acayen
 
Let general education 9
Let general education 9Let general education 9
Let general education 9Alex Acayen
 
5 tq gen ed
5 tq gen ed5 tq gen ed
5 tq gen edArneyo
 
5 tq gen ed (1)
5 tq gen ed (1)5 tq gen ed (1)
5 tq gen ed (1)Arneyo
 
Let questions (3)
Let questions (3)Let questions (3)
Let questions (3)Arneyo
 

Similar to Final coaching questions in general education by prof. cris paner (20)

Copy of 1.pdf
Copy of 1.pdfCopy of 1.pdf
Copy of 1.pdf
 
General education - 1.docx
General education - 1.docxGeneral education - 1.docx
General education - 1.docx
 
General education - 1.docx
General education - 1.docxGeneral education - 1.docx
General education - 1.docx
 
Gen-Ed-September-2016.docx
Gen-Ed-September-2016.docxGen-Ed-September-2016.docx
Gen-Ed-September-2016.docx
 
General education set a
General education set aGeneral education set a
General education set a
 
General education set a with highlighted answers)
General education set a with highlighted answers)General education set a with highlighted answers)
General education set a with highlighted answers)
 
General Education Drills
General Education DrillsGeneral Education Drills
General Education Drills
 
UNIFIED PRE-TEST IN INTRODUCTION TO THE PHILOSOPHY PPT.pptx
UNIFIED PRE-TEST IN INTRODUCTION TO THE PHILOSOPHY PPT.pptxUNIFIED PRE-TEST IN INTRODUCTION TO THE PHILOSOPHY PPT.pptx
UNIFIED PRE-TEST IN INTRODUCTION TO THE PHILOSOPHY PPT.pptx
 
Let general education 10
Let general education 10Let general education 10
Let general education 10
 
Gened science
Gened scienceGened science
Gened science
 
Let general education 2
Let general education 2Let general education 2
Let general education 2
 
Let general education 7
Let general education 7Let general education 7
Let general education 7
 
Let professional education 10
Let professional education 10Let professional education 10
Let professional education 10
 
2016 20 let_20bse_20pre-board_20with_20answer
2016 20 let_20bse_20pre-board_20with_20answer2016 20 let_20bse_20pre-board_20with_20answer
2016 20 let_20bse_20pre-board_20with_20answer
 
Let professional education 4.2
Let professional education 4.2Let professional education 4.2
Let professional education 4.2
 
Let general education 9
Let general education 9Let general education 9
Let general education 9
 
5 tq gen ed
5 tq gen ed5 tq gen ed
5 tq gen ed
 
5 tq gen ed (1)
5 tq gen ed (1)5 tq gen ed (1)
5 tq gen ed (1)
 
FINAL-COACHING-2019.pptx
FINAL-COACHING-2019.pptxFINAL-COACHING-2019.pptx
FINAL-COACHING-2019.pptx
 
Let questions (3)
Let questions (3)Let questions (3)
Let questions (3)
 

More from Arneyo

Consequences of plagiarism
Consequences of plagiarismConsequences of plagiarism
Consequences of plagiarismArneyo
 
Scandalous
ScandalousScandalous
ScandalousArneyo
 
Plagiarism
PlagiarismPlagiarism
PlagiarismArneyo
 
What is-an-organization-pp-for-report
What is-an-organization-pp-for-reportWhat is-an-organization-pp-for-report
What is-an-organization-pp-for-reportArneyo
 
Models of-organizational-behavior
Models of-organizational-behaviorModels of-organizational-behavior
Models of-organizational-behaviorArneyo
 
Human behavior report
Human behavior reportHuman behavior report
Human behavior reportArneyo
 
Decision making policy making and policy analysis
Decision making policy making and policy analysisDecision making policy making and policy analysis
Decision making policy making and policy analysisArneyo
 
Marungko
Marungko Marungko
Marungko Arneyo
 
Marungko file
Marungko fileMarungko file
Marungko fileArneyo
 
Basahon weekly mtb
Basahon weekly mtbBasahon weekly mtb
Basahon weekly mtbArneyo
 
Ms excel basic command
Ms excel basic commandMs excel basic command
Ms excel basic commandArneyo
 
The electronic class record user manual for grades 1 3 (self-contained class)
The electronic class record user manual for grades 1 3 (self-contained class)The electronic class record user manual for grades 1 3 (self-contained class)
The electronic class record user manual for grades 1 3 (self-contained class)Arneyo
 
Module9.ppst4.5.2
Module9.ppst4.5.2Module9.ppst4.5.2
Module9.ppst4.5.2Arneyo
 
Module8.ppst4.4.2
Module8.ppst4.4.2Module8.ppst4.4.2
Module8.ppst4.4.2Arneyo
 
Module7.ppst4.1.2
Module7.ppst4.1.2Module7.ppst4.1.2
Module7.ppst4.1.2Arneyo
 
Module6.ppst3.1.2
Module6.ppst3.1.2Module6.ppst3.1.2
Module6.ppst3.1.2Arneyo
 
Module12.ppst5.4.2
Module12.ppst5.4.2Module12.ppst5.4.2
Module12.ppst5.4.2Arneyo
 
Module11.ppst5.2.2
Module11.ppst5.2.2Module11.ppst5.2.2
Module11.ppst5.2.2Arneyo
 

More from Arneyo (20)

Consequences of plagiarism
Consequences of plagiarismConsequences of plagiarism
Consequences of plagiarism
 
Scandalous
ScandalousScandalous
Scandalous
 
Plagiarism
PlagiarismPlagiarism
Plagiarism
 
What is-an-organization-pp-for-report
What is-an-organization-pp-for-reportWhat is-an-organization-pp-for-report
What is-an-organization-pp-for-report
 
OB
OBOB
OB
 
Models of-organizational-behavior
Models of-organizational-behaviorModels of-organizational-behavior
Models of-organizational-behavior
 
Human behavior report
Human behavior reportHuman behavior report
Human behavior report
 
Ob
ObOb
Ob
 
Decision making policy making and policy analysis
Decision making policy making and policy analysisDecision making policy making and policy analysis
Decision making policy making and policy analysis
 
Marungko
Marungko Marungko
Marungko
 
Marungko file
Marungko fileMarungko file
Marungko file
 
Basahon weekly mtb
Basahon weekly mtbBasahon weekly mtb
Basahon weekly mtb
 
Ms excel basic command
Ms excel basic commandMs excel basic command
Ms excel basic command
 
The electronic class record user manual for grades 1 3 (self-contained class)
The electronic class record user manual for grades 1 3 (self-contained class)The electronic class record user manual for grades 1 3 (self-contained class)
The electronic class record user manual for grades 1 3 (self-contained class)
 
Module9.ppst4.5.2
Module9.ppst4.5.2Module9.ppst4.5.2
Module9.ppst4.5.2
 
Module8.ppst4.4.2
Module8.ppst4.4.2Module8.ppst4.4.2
Module8.ppst4.4.2
 
Module7.ppst4.1.2
Module7.ppst4.1.2Module7.ppst4.1.2
Module7.ppst4.1.2
 
Module6.ppst3.1.2
Module6.ppst3.1.2Module6.ppst3.1.2
Module6.ppst3.1.2
 
Module12.ppst5.4.2
Module12.ppst5.4.2Module12.ppst5.4.2
Module12.ppst5.4.2
 
Module11.ppst5.2.2
Module11.ppst5.2.2Module11.ppst5.2.2
Module11.ppst5.2.2
 

Recently uploaded

Difference Between Search & Browse Methods in Odoo 17
Difference Between Search & Browse Methods in Odoo 17Difference Between Search & Browse Methods in Odoo 17
Difference Between Search & Browse Methods in Odoo 17Celine George
 
Grade 9 Q4-MELC1-Active and Passive Voice.pptx
Grade 9 Q4-MELC1-Active and Passive Voice.pptxGrade 9 Q4-MELC1-Active and Passive Voice.pptx
Grade 9 Q4-MELC1-Active and Passive Voice.pptxChelloAnnAsuncion2
 
How to do quick user assign in kanban in Odoo 17 ERP
How to do quick user assign in kanban in Odoo 17 ERPHow to do quick user assign in kanban in Odoo 17 ERP
How to do quick user assign in kanban in Odoo 17 ERPCeline George
 
How to Configure Email Server in Odoo 17
How to Configure Email Server in Odoo 17How to Configure Email Server in Odoo 17
How to Configure Email Server in Odoo 17Celine George
 
MULTIDISCIPLINRY NATURE OF THE ENVIRONMENTAL STUDIES.pptx
MULTIDISCIPLINRY NATURE OF THE ENVIRONMENTAL STUDIES.pptxMULTIDISCIPLINRY NATURE OF THE ENVIRONMENTAL STUDIES.pptx
MULTIDISCIPLINRY NATURE OF THE ENVIRONMENTAL STUDIES.pptxAnupkumar Sharma
 
Solving Puzzles Benefits Everyone (English).pptx
Solving Puzzles Benefits Everyone (English).pptxSolving Puzzles Benefits Everyone (English).pptx
Solving Puzzles Benefits Everyone (English).pptxOH TEIK BIN
 
AMERICAN LANGUAGE HUB_Level2_Student'sBook_Answerkey.pdf
AMERICAN LANGUAGE HUB_Level2_Student'sBook_Answerkey.pdfAMERICAN LANGUAGE HUB_Level2_Student'sBook_Answerkey.pdf
AMERICAN LANGUAGE HUB_Level2_Student'sBook_Answerkey.pdfphamnguyenenglishnb
 
HỌC TỐT TIẾNG ANH 11 THEO CHƯƠNG TRÌNH GLOBAL SUCCESS ĐÁP ÁN CHI TIẾT - CẢ NĂ...
HỌC TỐT TIẾNG ANH 11 THEO CHƯƠNG TRÌNH GLOBAL SUCCESS ĐÁP ÁN CHI TIẾT - CẢ NĂ...HỌC TỐT TIẾNG ANH 11 THEO CHƯƠNG TRÌNH GLOBAL SUCCESS ĐÁP ÁN CHI TIẾT - CẢ NĂ...
HỌC TỐT TIẾNG ANH 11 THEO CHƯƠNG TRÌNH GLOBAL SUCCESS ĐÁP ÁN CHI TIẾT - CẢ NĂ...Nguyen Thanh Tu Collection
 
Framing an Appropriate Research Question 6b9b26d93da94caf993c038d9efcdedb.pdf
Framing an Appropriate Research Question 6b9b26d93da94caf993c038d9efcdedb.pdfFraming an Appropriate Research Question 6b9b26d93da94caf993c038d9efcdedb.pdf
Framing an Appropriate Research Question 6b9b26d93da94caf993c038d9efcdedb.pdfUjwalaBharambe
 
Types of Journalistic Writing Grade 8.pptx
Types of Journalistic Writing Grade 8.pptxTypes of Journalistic Writing Grade 8.pptx
Types of Journalistic Writing Grade 8.pptxEyham Joco
 
Computed Fields and api Depends in the Odoo 17
Computed Fields and api Depends in the Odoo 17Computed Fields and api Depends in the Odoo 17
Computed Fields and api Depends in the Odoo 17Celine George
 
ACC 2024 Chronicles. Cardiology. Exam.pdf
ACC 2024 Chronicles. Cardiology. Exam.pdfACC 2024 Chronicles. Cardiology. Exam.pdf
ACC 2024 Chronicles. Cardiology. Exam.pdfSpandanaRallapalli
 
AmericanHighSchoolsprezentacijaoskolama.
AmericanHighSchoolsprezentacijaoskolama.AmericanHighSchoolsprezentacijaoskolama.
AmericanHighSchoolsprezentacijaoskolama.arsicmarija21
 
ECONOMIC CONTEXT - PAPER 1 Q3: NEWSPAPERS.pptx
ECONOMIC CONTEXT - PAPER 1 Q3: NEWSPAPERS.pptxECONOMIC CONTEXT - PAPER 1 Q3: NEWSPAPERS.pptx
ECONOMIC CONTEXT - PAPER 1 Q3: NEWSPAPERS.pptxiammrhaywood
 
Employee wellbeing at the workplace.pptx
Employee wellbeing at the workplace.pptxEmployee wellbeing at the workplace.pptx
Employee wellbeing at the workplace.pptxNirmalaLoungPoorunde1
 
Introduction to ArtificiaI Intelligence in Higher Education
Introduction to ArtificiaI Intelligence in Higher EducationIntroduction to ArtificiaI Intelligence in Higher Education
Introduction to ArtificiaI Intelligence in Higher Educationpboyjonauth
 
Procuring digital preservation CAN be quick and painless with our new dynamic...
Procuring digital preservation CAN be quick and painless with our new dynamic...Procuring digital preservation CAN be quick and painless with our new dynamic...
Procuring digital preservation CAN be quick and painless with our new dynamic...Jisc
 

Recently uploaded (20)

Difference Between Search & Browse Methods in Odoo 17
Difference Between Search & Browse Methods in Odoo 17Difference Between Search & Browse Methods in Odoo 17
Difference Between Search & Browse Methods in Odoo 17
 
Grade 9 Q4-MELC1-Active and Passive Voice.pptx
Grade 9 Q4-MELC1-Active and Passive Voice.pptxGrade 9 Q4-MELC1-Active and Passive Voice.pptx
Grade 9 Q4-MELC1-Active and Passive Voice.pptx
 
How to do quick user assign in kanban in Odoo 17 ERP
How to do quick user assign in kanban in Odoo 17 ERPHow to do quick user assign in kanban in Odoo 17 ERP
How to do quick user assign in kanban in Odoo 17 ERP
 
How to Configure Email Server in Odoo 17
How to Configure Email Server in Odoo 17How to Configure Email Server in Odoo 17
How to Configure Email Server in Odoo 17
 
MULTIDISCIPLINRY NATURE OF THE ENVIRONMENTAL STUDIES.pptx
MULTIDISCIPLINRY NATURE OF THE ENVIRONMENTAL STUDIES.pptxMULTIDISCIPLINRY NATURE OF THE ENVIRONMENTAL STUDIES.pptx
MULTIDISCIPLINRY NATURE OF THE ENVIRONMENTAL STUDIES.pptx
 
Solving Puzzles Benefits Everyone (English).pptx
Solving Puzzles Benefits Everyone (English).pptxSolving Puzzles Benefits Everyone (English).pptx
Solving Puzzles Benefits Everyone (English).pptx
 
Rapple "Scholarly Communications and the Sustainable Development Goals"
Rapple "Scholarly Communications and the Sustainable Development Goals"Rapple "Scholarly Communications and the Sustainable Development Goals"
Rapple "Scholarly Communications and the Sustainable Development Goals"
 
Model Call Girl in Bikash Puri Delhi reach out to us at 🔝9953056974🔝
Model Call Girl in Bikash Puri  Delhi reach out to us at 🔝9953056974🔝Model Call Girl in Bikash Puri  Delhi reach out to us at 🔝9953056974🔝
Model Call Girl in Bikash Puri Delhi reach out to us at 🔝9953056974🔝
 
OS-operating systems- ch04 (Threads) ...
OS-operating systems- ch04 (Threads) ...OS-operating systems- ch04 (Threads) ...
OS-operating systems- ch04 (Threads) ...
 
AMERICAN LANGUAGE HUB_Level2_Student'sBook_Answerkey.pdf
AMERICAN LANGUAGE HUB_Level2_Student'sBook_Answerkey.pdfAMERICAN LANGUAGE HUB_Level2_Student'sBook_Answerkey.pdf
AMERICAN LANGUAGE HUB_Level2_Student'sBook_Answerkey.pdf
 
HỌC TỐT TIẾNG ANH 11 THEO CHƯƠNG TRÌNH GLOBAL SUCCESS ĐÁP ÁN CHI TIẾT - CẢ NĂ...
HỌC TỐT TIẾNG ANH 11 THEO CHƯƠNG TRÌNH GLOBAL SUCCESS ĐÁP ÁN CHI TIẾT - CẢ NĂ...HỌC TỐT TIẾNG ANH 11 THEO CHƯƠNG TRÌNH GLOBAL SUCCESS ĐÁP ÁN CHI TIẾT - CẢ NĂ...
HỌC TỐT TIẾNG ANH 11 THEO CHƯƠNG TRÌNH GLOBAL SUCCESS ĐÁP ÁN CHI TIẾT - CẢ NĂ...
 
Framing an Appropriate Research Question 6b9b26d93da94caf993c038d9efcdedb.pdf
Framing an Appropriate Research Question 6b9b26d93da94caf993c038d9efcdedb.pdfFraming an Appropriate Research Question 6b9b26d93da94caf993c038d9efcdedb.pdf
Framing an Appropriate Research Question 6b9b26d93da94caf993c038d9efcdedb.pdf
 
Types of Journalistic Writing Grade 8.pptx
Types of Journalistic Writing Grade 8.pptxTypes of Journalistic Writing Grade 8.pptx
Types of Journalistic Writing Grade 8.pptx
 
Computed Fields and api Depends in the Odoo 17
Computed Fields and api Depends in the Odoo 17Computed Fields and api Depends in the Odoo 17
Computed Fields and api Depends in the Odoo 17
 
ACC 2024 Chronicles. Cardiology. Exam.pdf
ACC 2024 Chronicles. Cardiology. Exam.pdfACC 2024 Chronicles. Cardiology. Exam.pdf
ACC 2024 Chronicles. Cardiology. Exam.pdf
 
AmericanHighSchoolsprezentacijaoskolama.
AmericanHighSchoolsprezentacijaoskolama.AmericanHighSchoolsprezentacijaoskolama.
AmericanHighSchoolsprezentacijaoskolama.
 
ECONOMIC CONTEXT - PAPER 1 Q3: NEWSPAPERS.pptx
ECONOMIC CONTEXT - PAPER 1 Q3: NEWSPAPERS.pptxECONOMIC CONTEXT - PAPER 1 Q3: NEWSPAPERS.pptx
ECONOMIC CONTEXT - PAPER 1 Q3: NEWSPAPERS.pptx
 
Employee wellbeing at the workplace.pptx
Employee wellbeing at the workplace.pptxEmployee wellbeing at the workplace.pptx
Employee wellbeing at the workplace.pptx
 
Introduction to ArtificiaI Intelligence in Higher Education
Introduction to ArtificiaI Intelligence in Higher EducationIntroduction to ArtificiaI Intelligence in Higher Education
Introduction to ArtificiaI Intelligence in Higher Education
 
Procuring digital preservation CAN be quick and painless with our new dynamic...
Procuring digital preservation CAN be quick and painless with our new dynamic...Procuring digital preservation CAN be quick and painless with our new dynamic...
Procuring digital preservation CAN be quick and painless with our new dynamic...
 

Final coaching questions in general education by prof. cris paner

  • 1. FINAL COACHING QUESTIONS IN GENERAL EDUCATION-200 ITEMS BY PROF. CRISENCIO M. PANER (LET TOPNOTCHER & VETERAN REVIEWER) 1. Cris Paner answered correctly 3/5 of the test. How many percent did she miss? A. 60% C. 30% B. 50% D. 40% 2.An angle is 14° more than its complement. Find the two angles. A. 45°& 45° C. 30°& 60° B. 52°& 38° D. 55°& 125° 3. Evaluate 30 + 3-2 ( 34 ) A. 81 C. 10 B. 9 D. 27 4. A Gynecologist uses 1 ½ hours to treat a patient. She usually works 12 hours a day, from Monday to Saturday. How many patients can she treat in one week? A. 16 C. 48 B. 8 D. 36 5. If 15 is subtracted from three times a certain number the result is 27. Find the number. A. 14 C. 81 B. 9 D. 20 Situational Maraming katulad ang buhay ng tao. Ito’y tulad ng putahe na “sweet sour dish.” May matamis at maasim na lasa. Katulad din ito ng panahon, may mabagyo at may maaraw. Nakararanas tayo ng malulungkot na pangyayari at kung minsan ay masasayang karanasan. Kainlangan ay lagi tayong handa. Anuman ang pangyayaring dumating ay dapat nating harapin ng buong tapang, maging ito’y mapait na karanasan. Kung mabagyo ang buhay, lagi nating sabihin ang ganito: Kung kaya nila itong nararanasan ko, kakayanin ko rin. Hindi ito ibibigay ng Panginoon kung hindi ko kaya. Nagtitiwala ako sa kanta dahil hindi Niya ako pinapabayaan. 6. Ano ang mahalagang pag-uugali ng tao ang pinahihiwatig ng binasang talataan? A. Maging makatarungan tayo sa lahat ng bagay, tao at panahon. B. Humanda tayong lagi sa mga suliraning darating sa ating buhay. C. Maging positibo tayo sa pag-iisip at pagbibigay pasiya. D. Gawin natin lagi ang magagandang bagay, ang pagtawag at paghingi ng tulong sa Diyos. 7. Saan itinulad ng may-akda an gating buhay? A. Sa malakas na bagyo B. Sa masarap na putahe C. Sa init at lamig ng panahon
  • 2. D. Sa putahe at sa panahon 8. Ano ang pangunahing paksa na inilahad sa talataan? A. Harapin ang buhay at isaisip ang kasabihang umasa sa Diyos na hindi magpapabaya. B. Pagkat kaloob ng Diyos sa tao ang pagdurusa, sakit at suliranin, nariyan Siya upang tao ay damayan. C. Harapin nang buong tapang ang buhay na may lungkot at ligaya, at magtiwala sa Poong Maykapal. D. Buong giting na humarap sa buhay mabagyo man o maaraw. 9. Ano ang nagbunsod ng pagbabago sa yugto ng panitikang Filipino? A.Pagdating ng Amerikano sa bansa B.Pag-aabuso ng kastilasa mamamayan C.Pag-alis ng mga tao da kawit D.Panggunita kina Padre Gomez, Burgos at Zamora 10.Lack of circulation and oxygen caused in child to die of: A.asphyxiation C. pneumonia B.heart failure D. suffocation 11. Fish in aquarium all died when a drop of ____ was placed in the water. A.alcohol C.sulphur B.soap D.sugar 12. Children with poor eye sights were advised to improve their nutrition through this were advised to improved their nutrition through this combination of food: A.pork, bread, potato B.rice, squash, fish C.meat, rice, ice cream D.hamburger, french fries, coke 13. Water is sold in bottles. The best purifying process is done through: A.Osmosis C. Boiling B.Purification D. Distillation 14.Children in upland places are found affected by enlarged goiter. Which of these situations may help these group of children? A.Add iodized salt to diet B.Add iron to diet C.Expose the children to ultraviolet rays D.Add iodine drops to diet 15.The urbanization of municipalities have added delays in the Agrarian Reform efforts, because _____. A.Limited land area resulted in converting public land to subdivisions B.Agricultural lands were converted to residential areas C.Coordination of DAR, DA, AND DILG is met D.Agricultural lands were sold to industrial establishments 16.The appointment of cabinet members was recalled because ___________. A.The official had no clearance
  • 3. B.The resistance was strong C.The official was incompetent D.The appointing officer was mislead 17.Rural families do not believe in limiting the number of children since they argue: A.A small family is not ideal for farmers who own animals and work on wide track of land B.A big family is always happy and economically advantageous C.Having more children means more helping hands D.God does the limiting of children not man 18. A young girl left her baby at the household of a monastery. The child grew up without knowing her mother and father. What right is deprived of the child? A.Right to assert individuality B.Right to freedom of choice C.Right to life D.Right to quality education 19.No country will progress unless he is a part of a world organization and its agreement. What is referred to? A.UNESCO B.NATO C.WHO D.GATT 20. Which is the BEST WAY to write the underlined portion of this sentence? However, their VCR kept them from missing their favorite primetime shows. A. keepes B. keeps C. hard kept D. keeped 21. Which is the BEST WAY to write the underlined portion of this sentence? A person should keep in mind some basic safety rules when you are deciding whether or not to use a fire extinguisher. A. rules you decide B. rules when you decided C. rules you are deciding D. rules when deciding 22. What correction should be made to this sentence? Most State tourism departments and some travel agencies have bed and breakfast listings. A. Insert a comma after agencies B. Change have to has C. Change tourism to Tourism D. Change State to state 23.What correction should be made to this sentence? Recently, educators exammined the effectiveness of computer instruction in schools. A. Replace educators with educator's B. Change the spelling of exammined to examined C. Change schools to Schools
  • 4. D. Replace computer with computer's 24. Which is the BEST WAY to write the underlined portion of this sentence? However, if a fire extinguisher is handy, a quick-thinking person often can use them to put out a small fire. A. They B. Him C. Them D. lt 25.Anong dulog pampanitikan ang kilala rin sa tawag bilang reader-response theory? A. Antropolohiya B. Impresyonista C. Patalambuhay D. Pansikolohiya 26. Ano ang tinataglay ng mga sumusunod na salita: tanaw, aliw, kamay, reyna? A. Diptonggo B. Pares minimal C. Klaster D. Ponema 27. What do you call the molecule that contains the genetic information of the organism? A. Nucleic Acid B. Nucleolus C. Ribosomes D. Organelles 28. Which power of the state enables it to impose charge of burden upon persons, property or property rights for the use and support of the government expenditures for social services and a way of revenue collection? A. Eminent domain B. Expropriation C. Value added tax D. Taxation 29. What was the first term given by Marcelo H. Del Pilar to the notorious invisible influence and domination by Spanish religious priests over the colonial government. A. Lassuert partidas B. Pase Region C. Frailo cracia D. Complace 30. The fundamental right invoked by filing the “Writ of Amparo” is _____ A. The right to self defense
  • 5. B. The right to due process C. The right to life, liberty and security D. The right to be defended by a public attorney 31. Which of the following is considered the lowest form of learning? A. Perceiving B. Responding C. Conditioning D. Teaching 32. Reason must be used in understanding the existence of God. Who advocated this philosophy? A. St. Benedict B. St. Peter C. St. John D. St. Thomas Aquinas 33. If the principles and theories of human behavior were to be applied to teaching and learning. The field will be called ______. A. Educational Theory B. Educational Philosophy C. Educational Psychology D. Educational Sociology 34. In July 1901, Isabelo delos Reyes founded the first labor union in the country. What was its name? A. Association of the Philippine Labor B. Union Obrera Democratica C. Union Trabajadores de Filipinos D. Association De Compania Tabacalera 35. The Philippine lies in the _____, an area where volcanoes are active. A. Archipelagic fault line B. Ring of fire C. Wheel of fire D. Volcanic Rim 36. During the June 12, 1898 Declaration of Independence, a band played the Marcha Nacional Filipino What band was this? A. Pangkatkawayang ng Pateros B. Kawit Cavite Band C. Malabon Band D. San Francisco de Malabon 37. This is modern technology’s response to message previously sent over couriers or post offices A. E-shopping B. E-Registry C. E-Mail D. Frailocracia
  • 6. 38. What is the case study method of research? A. An intensive study of one individual. Typically, this may involve interviews, observation, experiments and tests. B. It involves the acquisition, synthesis, organization and presentation of information. C. The research manipulates variables, randomly assign participants to various conditions and seek to control other influences. D. Involves going out and asking a questions about the phenomenon of interest. 39. What is the topical method of research? A. An intensive study of one individual. Typically, this may involve interviews, observation, experiments and tests B. It involves the acquisition, synthesis, organization and presentation of information. C. The research manipulates variables, randomly assign participants to various conditions and seek to control other influences. D. Involves going out and asking a questions about the phenomenon of interest. 40. Not many students are aware that Jose Rizal, the Philippine's National Hero is also a sculptor. Which among the following is a sculpture of Rizal? A. The Oblation B. The Power of Science over Death C. Lapu-Lapu Monument D. The Philippine Map Relief 41. Teacher Cris Paner, BSED graduate resigns in her current teaching position as high school teacher because she has been force to teach in preschool department by the Principal in this school year. Teacher Jenny's voluntary resignation resulted to? A. Structural Unemployment B. Cyclical Unemployment C. Frictional Unemployment D. Unemployment 42. The Comprehensive Agrarian Reform Law popular as CARL is the lawful basis of the full Implementation of the Comprehensive Agrarian Reform Program (CARP). This was during the administration of? A. Fidel V. Ramos B. Ferdinand Marcos C. Corazon Aquino D. Joseph Estrada 43. How is coral atool formed? A. Volcanic eruption B. Corals growing around a volcanic island C. Underwater bedrock formations D. Earthquake 44. What is a long shore drift? A. Movement of sand and shingles along the coast B. Sand bars
  • 7. C. Accumulation of sad at the river mouth D. Island formed by volcanic eruptions 45. How does an occlusion form? A. Cold air moving up from the ground B. Cold front pushing warm air up of the ground C. Unbalance electrical reaction in the air D. Cold and warm air mixing in the atmosphere 46. What is a heat haze? A. A reflection caused by pollutants in the air B. A distorted image resulting from the bending o sun’s light rays by changes in air temperature C. A movement o warm air over a vast expanse of land D. Caused by extremely high temperature common in dessert areas 47. What sort of rock formation do the world’s greatest mountain ranges consist of? A. Magma C. Fold eruptions B. Chalk deposit D. Slip formation 48. What is the force that wears down mountains? A. Earthquake C. Volcanic eruptions B. Erosion D. Deforestation 49. How are volcanic islands formed? A. Collision of two oceanic plates B. Cooling of lava by seawater C. Volcanic eruptions D. Accumulation of corals 50. Ang may-akda ng “Kahapon ,Ngayon at Bukas” ay si ________. A. Aurelio Tolentino B. Juan Abad C. Alejandro Abadilla D. Severino Reyes 51. Isang awiting bayan na tungkol sa paglilibing ang _______. A. umbay B. kundiman C. sambotani D.soliranin 52. The highest mountain in the continent of Africa is ______ . A. Mt.Everest B. Mt.Kilimanjaro C. Mt. Fuji D. Mt. Apo 53. The “yellow race “ is also known as : A. Caucasians B. Mongoloids C.Negroids D.Malays 54. The function by which schools help children to participate effectively in larger society is called : A. Socialization B. Education C. Enculturation D. Acculturation 55. What phase of the Moon is observed when it cannot be seen at all because it passes directly between the earth and the Sun ? A. New Moon B. First quarter C. Full Moon D. Last quarter
  • 8. 56. May sagisag panulat na Paralitiko at ang tinaguriang “Utak ng Himagsikan” , ay si _______. A. Emilio Jacinto B. Antonio Luna C. Jose Corazon de Jesus D. Apolinario Mabini Below is a Biographical sketch of an American movies writer: How Did Elvis Presley Achieve Recognition Success often comes to those with humble beginnings, Elvis Aaron Presley was born on January 8, 1935, in Tupelo, Mississippi. He first sang in a church choir and taught himself to play the guitar, but he never learned to read music. By 1953, he had moved to Memphis, Tennessee, graduated from high school, and enrolled in night school to become an electrician. That year, at Sun Records, Presley recorded a personal record for his mother, a song that was heard by the company's president. As a result of the president's recognition, Presley's first record, "That's all Right, Mama," was out in 1954. He toured the South, and in 1955 five of his records were released simultaneously. His first national television appearance was that year on Jackie Gleason's "The Stage Show," but Presley became known for his appearance on "The Ed Sullivan Show," where the young singer gyrated as he sang "rock n' roll" music. During the live television performance, Presley was photographed only from the waist up because his motions were considered obscene. "Elvis the Pelvis" began his film career in 1956 with LOVE ME TENDER and signed a long term film contract. The movie critics were not always kind, but teenagers flocked to Presley's films. Within a few short years, Presley had established a career that would span twenty-five years of ups and downs and make him one of the most popular entertainers in history. Long after his untimely death at age 42, Presley would be remembered as "The King of Rock n' Roll” 57. The statement that "success often comes to those with humble beginnings" would apply best to which of the following figures? A. Ramon Magsaysay B. Gloria M. Arroyo C. Corazon C. Aquino D. Joseph E. Estrada 58. The main idea of the sketch is that A. singers are more successful if they appear in films B. there has always been obscenity on television C. opportunity and luck are often as important as hard work D. celebrities are usually more famous after their death 59. The last sentence reveals that the author's attitude toward Presley is one of A. indifference B. admiration C. disbelief D. disgust The poem below is entitled "Suburban Prophecy" which is written by Howard Nemerov. On Saturday, the power-mowers' whine
  • 9. Begins the morning. Over this neighbourhood Rises the keening, petulant voice, begin Green oily teeth to chatter and munch the cud. Monsters, crawling the carpets of the world, Still send from underground against your blades The roots of things battalions green and curled And tender, that will match your blades with blades Till the revolted throats shall strangle on . The tickle of their dead, till straws shall break Crankshafts like camels, and the sun go down On dinosaurs in swamps. A night attack Follows and by the time the Sabbath dawns All armored beasts are eaten by their lawns. 60. How long does the action of the poem take place? A. A week B. Twenty-four horns C. An afternoon D. A morning 61. The poet's use of words such as whine, voice, teeth, chatter and munch is to suggest that the power-mowers are A. very powerful B. alive C. like cows D. green 62. To what does the phrase "your blades" in line 8 refer? A. Lawnmowers B. Roots C. Carpets D. Monsters 63. The imagery in the first stanza appeals to the reader's sense of A. sight B. touch C. smell D. hearing Ang sumusunod ay isang talumpati na may pamagat na SA KABATAAN na sinulat ni Onofre Pagsanghan. Isa sa mga salitang napag-aralan natin sa wikang Pilipino ay ang salitang NABANSOT. Kapang ang isang bagay daw ay dapat pang lumaki ngunit ito'y tumigil na sa paglaki, ang bagay na ito raw ay NABANSOT. Marami raw uri ng pagkabansot ngunit ay pinakamalungkot na uri raw ay ang pagkabansot ng isipan, ng puso at ng diwa.
  • 10. Ang panahon ng kabataan ay panahon ng paglaki, ngunit ang ating paglaki ay kailangang paglaki, at pag-unlad ng ating buong katauhan, hindi lamang ng ating sukat at timbang. Kung ga-poste man ang ating taas at ga-pison man ang ating bigat ngunit kung ang pag-iisip naman nati'y ga-kulisap lamang kay pangit na kabansutan. Kung tumangkad man tayong tangkad-kawayan at bumilog man tayong bilog- tapayan, ngunit kung tayo nama'y tulad ni "bondying" ay di mapagkatiwalaan-anong laking kakulangan. Kung magkakatawan tayong katawang "Tarzan" at mapatalas ang ating isipang sintalas ng kay Rizal, ngunit kung ang ating kalooban nama'y itim na duwende ng kasamaan-anong kapinsalaan para sa kinabukasan. Kinabukasan, kabataan, tayo raw ang pag-asa ng Inang Bayan. Tayo raw ang maghahatid sa kanya sa langit ng kasaganaan at karangalan o hihils sa kanya sa putik ng kahirapan at kahihiyan. Ang panahon ng pagkilos ay ngayon, hindi bukas, hindi sa isang taon. Araw-araw ay tumutuwid tayong palangit or bumabaluktod tayong paputik. Tamang-tama ang sabi ng ating mga ninunong kung ano raw ang kinamihasnan ay siyang pagkakatandaan. Huwag nating akalaing makapagpapabaya tayo ng ating pag-aaral ngayon at sa araw ng bukas ay bigla tayong magiging mga dalubhasang magpapaunlad sa bayan. Huwag nating akalaing makapaglulublob tayo ngayon sa kalaswaaan at kahalayan at sa mahiwagang araw ng bukas sigla tayong magiging ulirang mga magulang. Kabataan, tunay na pag-ibig sa bayan, ang tunay na nasyonalismo, ay wala sa tamis ng pangarap wala rin sa pagpag ng dila. Ang tunay na pag-ibig ay nasa pawis ng gawa. 64.Sa alin makikita ang tunay na NASYONALlSMO? A. Diwa B. Gawa C. Sulat D. Salita 65. Alin salita ang paulit-ulit na binabanggit ni Onofre Pagsanghan? A. Nabansot B. Bayan C. Kabataan D. Kung 66. Bakit di dapat tumangkad tulad ni "bondying"? A. Di ito magpakakatiwalaan B. Di totoo ito C. May kakulangan ito D. Magulo kasi ito 67. Alin sa mga sumusunod ang mensahe ng talumpati? A. Ang mataas na paniniwala at taimtim na pananalig ay kailangang taglayin upang ang hangarin sa buhay ay ating kamtin. B. Ang panahon ng kabataan ay panahon ng paglaki at pagbabagong makabuluhan. C. Ang gawa ang siyang sukat ng kadakilaan. D. Ang kabataan ay siyang pag-asa ng bayan.
  • 11. 68. What is meant by TWO PINS in this sentence? For two pins I could have hit him on the nose. A. A second course of action B. Without much persuading C. The second chance D. Have a second alternative choice 69. What is meant by AT SIXES AND SEVENS in this sentence? We moved into the house last week, but I'm afraid everything, is still at sixes and sevens. A. The things have not been shipped. B. In a state of confusion C. In an orderly manner D. The boxes are still intact 70. Which word ends with [S] pronounced [Z]? A. Maps B. Jakes C. Laughs D. Buys 71. What is meant by SOFT OPTION in these sentences? Rebecca realized that if she stayed in her present job it would mean competing with an envious rival. Leaving the company would probably be a soft option. A. An action that is difficult to take B. An action that is not agreeable C. An action that is easier. D. An action that is weakly funded 72. Which word contains the voiced Th? A. Thank B. These C. Think D. Thing 73. To gain the attention of the audience, the trick is __________? A. start low, speak hurriedly B. start high, speak loudly C. start high, speak rapidly D. start low, speak slowly 74. Which word contains the [ae] sound? A. Carriage B. Cabin C. Caste D. Can
  • 12. 75. Which of the following words DOES NOT contain the [voiceless th]? A. Mouth B. Breath C. Teeth D. Health 76. Which pitch is used for the word STRANGE? What a strange story! A. 3 B. 2 C. 4 D. 1 77. What do the following lines CONVEY? Midnight, not a sound from the pavement. Has the moon lost her memory? She is smiling alone. In the lamp light the withered leaves Collect at my feet And the wind begins to moan A. Confusion B. Optimism C. Loneliness D. Eagerness 78. I can’t find ________ calculator; may I use _________, Prince? A. My, yours B. His, yours C. My, his D. Yours, mine 79. Neither the teacher nor the students ________________ present in the affairs. A. Am B. Was C. Is D. Were 80. We admire ___________ when Cris Paner speaks English fluently with _______________. A. Him, them B. Her, us C. Him, we D. Him, me 81. “The figures must be TRANSMUTED in order to understand the grade.” The capitalized word means: A. Estimated B. Surpassed C. Changed
  • 13. D. Summed 82. Some preachers suggest the _____________ that the end of the world is near. A. Proposal B. Prophecy C. Prophetic D. Prophet 83. Filipino writer whose stories and poems depict Filipino-Spanish cultural beliefs and traditions. A. Edilberto Tiempo B. N.V.M. Gonzales C. Bienvenido Santos D. Nick Joaquin 84.The people of Montalban in Rizal raised protest on the landfill since. A.Water seepage would pollute water source B.People were disturbed by garbage trucks C.Air pollution could not be controlled D.Garbage cold be recycled 85.The weather report recorded a temperature of 29°C in the morning and 33.5°C at two O’clock in the afternoon. How many degrees higher was the afternoon temperature A. 4.5°C B. 5.8°C C. 6.3°C D. 5.5°C 86.Why are Filipino overseas workers considered modern time heroes? A.They sacrifice all-the family life and economies just to improve their quality of life. B.They became broad minded, skilled and economically stable. C.They have helped stabilize Philippine economy D.They come home ready to feast their townmales and friends 87.Hindi naman pangit ang larawan ngunit _____ang kanyang anyo. A. pinapuri – puri C. kapuri-purihan B. kapuri – puri D. kapurihan 88. I have one life to give and I give it al to my country.” This pictures a: A. a soldier at war B. worker in a cement factory C. a policeman on the street D. a doctor in the operating room. 89. The endangered species are protected by DENR. All these are concerns of: A. Tourism C. Urbanization B. Industrialization D. Terrorism 90. Abuses to soldiers, women, children and the helpless are attended to by a Commission. Who was the first Commissioner on Human Rights? A. Hon. Aurora Recine
  • 14. B. Hon. Mary Conception Bautista C. Hon. Teofisto Guingona D. Hon. Sedfrey Ordonez 91. After a series of tutorial lessons at PANER Tutorial Center, Sophia’s grade in Math rose from 78 to 88. By what percent did her average increase? A. 35.75% C. 11.36% B. 12.82% D. 20.16% 92. If four mangoes cost ₱29.00, at that price, what will 2 ½ dozen mangoes cost? A. ₱370.00 C. ₱188.50 B. ₱348.50 D. ₱217.50 93. A room of 10m by 7m. There is a 7.5m by 5m carpet in the middle. What percent of the room is uncovered? A. 46.4% C. 48% B. 53.6% D. 80% 94.One angle of a parallelogram is 35°. What are the measures of the three other angles? A. 145°, 35°, 145° C. 85°, 135°, 140° B. 45°, 65°, 170° D. 35°, 65°, 65° 95.If the scores of 10 students are 76, 80, 75, 83, 80, 79, 85, 80, 88, 90, the mode is___ A. 79 C. 80 B. 85 D. 88 96. Which of the following methods can all diabetics control their condition and avoid heart disease and blindness? I. Regulating their intake of glucose II. Increasing the levels of insulin in the body by taking insulin injections III. Maintaining a reasonable exercise regimen to keep weight down. A. I B. II C. I and II D. I and III 97. The principle under which a thermostat operates is the same when A. a gas expands to fill the container in which it is held. B. a pendulum swings when it is set into motion. C. a chemical reaction occurs when two substances combine. D. the level of mercury rises or falls in a glass tube. 98. Why does a bullet when discharged into the air eventually fall to the ground? This is due to the Law of A. Universal Gravitation B. Applied Force C. Inertia D. Action and Reaction 99.The stanza below is taken from "Barter" by Sara Teasdale. Life has loneliness to sell, Music like a curve of gold, Scent of pine trees in the rain, Eyes that love you, arms that hold,
  • 15. And for your spirit's still delight, Holy thoughts that star the night. To what does Teasdale compare music? A. The scent of pine trees B. A curve of gold C. Eyes that love D. The rain 100.An elderly woman suffered a stroke, a restriction of blood flow to the brain. If the stroke caused to the right side of her body to become temporarily paralyzed, she most likely experienced a decreased blood flow to A. the left side of her body. B. the front of her brain. C. the left side of her brain. D. the right side of her brain. 101.Which is the BEST WAY to write the underlined portion of this sentence? Many viewers taped shows to watch later. A. tapped B. had taped C. tape D. had tapped 102.Alin ang kahulugan ng KAHIRAMANG SUKLAY? A. Kakilala B. Kaibigan C. Karibal D. Kalahi 103.Ano ang pinakaangkop na kahulugan nito? Nagsasaya tayo ngayon sapagkat ang iyong namatay na kapatid ay muling nabuhay; ang nawawala ay muling nakita. A. Ang pagbabalik ay dapat ipaghanda nang malaki. B. Ang pagbabago ng kapatid ay dapat pahalagahan. C. Dapat silang magsaya sa muli nilang pagsasama-sama D. Ang pagsasama nila ay dahil sa muling pagbabalik ng kapatid. 104.Two days after Japan attacked Pearl Harbor, Roosevelt made the following statements: In the past few years and most violently in the past few days, we have learned a terrible lesson. We must begin the great task that is before us by abandoning once and for all the illusion that we can never again isolate ourselves from the rest of humanity. In the statement, Roosevelt is expressing the ideas of __________. A. an internationalist B. an anti-imperialist C. an imperialist D. an isolationist 105.Which pitch is used for the word STRANGE? What a strange story! A. 3 B. 2 C. 4 D. 1 106.What is the function of DIFFUSION in the human body? A. Regulates blood flow B. Plays an insignificant role in the body's functioning C. Allows an even distribution of substances throughout all cells of the body D. Comes into play in times of extreme illness 107.Utang sa kanyang sipag at sikap sa paggawa ang kanilang maalwang pamumuhay. A. Pangngalan B. Pangngalang-diwa C. Pawatas D. Pang-ukol 108.Which of the following represent, ethnocentric behavior?
  • 16. A. A tourist who lectures his foreign hosts on the "uncivilized" nature of their marriage customs B. A student who tutors an immigrant in English C. A Hispanic community group demands that public aid forms be published in English and Spanish. D. A Peace Corps volunteer who helps dig wells in Central Africa 109.The following are legitimate children, EXCEPT A. those born by artificial insemination. B. those legitimated. C. those born during a valid marriage of parents. D. those born out of a valid marriage of the parents. Write the meaning of the underlined idiom from the choices. 110. There's too much red ink in the company's financial statement. A. Financial loss C. Financial debt B. Financial gain D. Financial obligation 111. In doing things, you must always bear in mind this ''Rome was not built in a day" A. Accept and bear consequences of your own doing B. Hatred between forgiven two parties. C. Great things require time & effort D. Always on the move 112. Amiel is paddling on his own canoe because he wants to concentrate on his thesis. A. To work overtime C. To work well in groups B. To work without help D. To work with the help of someone else 113. Jona is a responsible sister. She always takes care of here siblings. In the sentence, she refers to Jona, so it is called the: A. Antecedent C. Predicate noun B. Reference D. Pronoun 114. _____ the floods that damaged their crops, the farmers are still hopeful of an abundant harvest. A. Spite of C. In spite B. Despite D. Despite of 115. Most teachers are familiar _____ the communicative language teaching approach. A. By C. With B. In D. Of 116. The teacher not the academic managers _____ to attend the workshop on Outcomes Based Education two days from now. A. Are going C. is going B. Were going D. Was going 117. All students must submit his homework tomorrow. A. Student C. His B. their D. Tomorrow
  • 17. 118. Let's go to the nearest store, isn’t it ? A. Let's go C. The B. Store D. Will we 119. ''A rabbit ran across the street with long ears". Which sentence error is exhibited in the sentence? A. Misplaced modifier C. Dangling modifier B. Run-on sentence D. Faulty diction 120. If I _____ the money, I would buy a yacht. A. Had C. Have B. Had had D. Have had 121. A. Then you will come to a hallway leading to the library's music room B. Walking around the information desk to the left, you will pass children's reading room C. That leads to the music room D. You enter the main door of the library, you see the information desk directly in front of you E. At the end of the hallway you will see a sign What is the proper sequence of the sentence? A. DBAEC C. BDAEC B. BDEAC D. DBEAC 122.Siya ay si “Dolores Manapat”. A. Graciano Lopez Jaena B. N.V.M. Gonzalez C. Andres Bonifacio D. Marcelo H. del Pilar 123. The process of comparing one quantity with corresponding standard is called ______ . A. Measurement B. Formulation C. Interpolation D. Extrapolation 124.Matter is anything that has ______. A. Weight and volume B. Mass and volume C. Weight and mass D. Mass and shape 125. A student walks from home to school 3 blocks away and then returns home for lunch. After lunch, he goes back to school. His total displacement is _______ . A. 0 block B. 3 block C. 6 blocks D. 9 blocks 126. What property of matter is involved in the sign, “Fasten Your Seat Belt”? A. Impenetrability B. Inertia C. Acceleration D. Interaction 127. Which of the following situations is true as a boat moves from salty sea to fresh water river? A. It sinks deeper. B. It floats higher. C. It stays at the same level. D. It moves faster. 128. Which of the following is conserved when a pendulum is swinging? A. Mass B. Momentum C. Energy D. Weight 129. Which statement best explains why ice is a good refrigerant ?
  • 18. A. It cools by evaporation. B.It absorbs heat while melting. C.It conducts heat quickly. D.It does not melt inside the refrigerator. 130. An object becomes positively charge if it _________ . A. gains protons B. gains electron C. loses protons D. loses electrons 131. The force that holds the atoms and molecules together to form larger bodies is known as A. Gravitational force B. Electromagnetic force C. Weak nuclear force D. Strong nuclear force 132. What is the maximum number of electrons that can be accommodated in n=4? A. 2 B. 8 C. 16 D. 32 133. It is characterized by shared experiences and mutual responsibilities. A.Gesellschaft B. Gemeinchaft C. Enculturation D. Acculturation 134. It features the exchange of goods, money and services . A. Gesellschaft B. Gemeinchaft C. Enculturation D. Acculturation 135. The longest revolt in the Philippine History is called the ______ . A. Dagohoy revolt B.Diego Silang revolt C. Gabriela Silang revolt D. Palaris revolt 136. Violation of human rights is a violation of the _____ of persons. A. dignity B. intelligence C. emotions D. freedom 137. Which of the following chronological order for the events in history listed below? I. Execution of Dr. Jose Rizal II. Declaration of the First Philippine Republic III. Edsa Revolution IV. World War II A. I, IV, II, III C. II, I, III, IV B. II, I, IV, III D. I, II, IV, III 138. Which primary group is considered the "nursery of human nature"? A. Play group C. Classmate's group B. Peer group D. Family group 139. One of the strengths of the Filipino character is "pakikipagkapwa-tao". This is manifested in one of the following except one: A. Pakikiramdam C. Pakikiramay B. Malasakit D. Lakas ng loob 140. In every society, certain position are assigned to individuals on the basis of age, sex, marital status, and similar to other criteria. These patterns are: A. University accepted and practice C. Taught in schools B. Learned from parents' D. Culturally defined
  • 19. 141. Which of the following action must be done to show perseverance if you failed in your project the first time you do it? A. Stop doing the project and try a much easier one B. Tell tour teacher you will not do it anymore C. Try and try until you produce one D. Ask somebody to do it for you 142. Which of the following scientific attitude is shown by the person who keeps seeking answer to things he is interested with? A. Curiosity C. Open mindedness B. Honesty D. Resourcefulness 143. Democracy refers to the involvement of all citizens in the control of government, while an oligarchy relegates control of the government to: A. An autocrat C. The mother country B. A select few D. Religious group 144. The benevolent assimilation proclamation on December 21, 1898, was the first official indication of American policy regarding the Philippines. It is expressly indicated the intention of the United States to stay in the Philippines by exercising the right of sovereignty over the Filipinos. Who was the President who issued this "Benevolent Assimilation" policy? A. Eisenhower C. McKinley B. Wilson D. Roosevelt 145. What is the social scientist's explanation of the relationship of a leader to history? A. Leaders are a product of their times B. History is a reflection of great leaders C. Every great movement is the lengthened shadow of a great man D. Had any great leader been different than he was, the course of would have been different 146. Which of the following forces of social change have the greatest impact upon the traditional Filipino family in the 20th century? A. Immigration and Migration B. Socialization and Stratification C. Specialization and Assimilation D. Industrialization and Urbanization 147. Through the process of socialization, the individual is able to: A. Function as contributing member of the group B. Learn to dialogue with peers and adult C. Satisfy his personal needs and drives D. Become an adult member of society 148. Emilio Jancinto's literary works were written in Tagalog and small part in Spanish. The best in all of his works was written o October 8, 1897. it was visibly an animation of Rizal's ''Mi Ultimo Adios”. What was the title of his poem? A. Mi Madre C. Ala Juventud Filipina B. Ala patria D. Amor Patrio
  • 20. 149. The committe recommends that she _____ the proposal on time. A. Submit C. Submitted B. Submits D. Will submit 150. If I were you, I _____ not be joining the contestant. A. Will C. Would B. Were D. Will be 151. We _____adhere to the school's policy that ''no down payment, no enrollment. A. Strict C. Strictly B. Strictness D. Strictest 152. It's a necessity, I _____ see my thesis adviser. A. Could C. May B. Shall D. Need to 153. if you had saved your money, you _____ your nephew to school. A. Would send C. would have send B. Send D. will send 154. The school director suggests that the suspended student _____ a written explanation. A. Makes C. Made B. Make D. Will make 155. I want to find out which is better _____ Superman, X-men and Spider man. A. Between C. Among B. From D. With 156. They _____ to Baguio for five hours when the hard rain fell. A. Had driven C. Have been driving B. Have driven D. Had been driving 157. She is fond _____ stamps and stationeries. A. By collecting C. For collecting B. In collecting D. Of collecting 158. The rich _____ the most powerful in the society A. Was C. Is B. Were D. Are 159. I think one or two of the guests' _____ here now. A. Are C.Was B. Were D. Is 160. Speaker 1: Where were you last night? Speaker 2: Last night? I am at Starbucks SM Manila. The speaker who committed an error is:
  • 21. A. Speaker 1 C. Speaker 2 B. Both speakers D. None of the above 161. The Nibelungenlied is a A. Latin Myth B. Chinese legend C. Russian folk song D. Medieval German epic 162. An association wherein the name of something is substituted by something that represents it. A. Metonymy B. Comparison C. Euphemism D. Personification 163. Because the moon rotates on its axis at the same time as it ______ around the earth, we see the same side A. Revolve B. Revolves C. Is revolving D. Has been revolving 164. In English verse, a poetic foot having 1 stressed syllable followed by 1 unstressed syllable is ______. A. Trochaic B. Iambic C. Dactylic D. Anapaestic 165. Senators were accused by activists of washing their hands with the perfumes of Arabia. This state is commonly known as A. Guilt B. Triumph C. Indecision D. Aggression 166. It is a collection of religious poetry written by Rabindranath Tagore. A. Mahabharata B. Gitanjali C. The Ramayana D. Bhagavad Gita 167. I suggest that he _____ in the room for one week. A. Stay B. Stayed C. Staying D. Stays
  • 22. 168. The manager told his workers, “We have to reduce our workforce.” What did he mean? A.Workers are free to leave B.Workers are warned of possible lay off C.Workers have to double time on their jobs D.Workers should submit themselves to a reducing gym 169. The parent remarked, “__________ I come late, just lock the door.” A. In the absence B. In the process C. In the event D. In the case 170. The copyreader found the news story boring. He found it full of _____. A. Adjectives B. Verbs C. Pronouns D. Adverbs 171. There were three guests on the stage. They were made up of a parent, the governor and the principal. Who should be acknowledged first by the valedictorian? A. The classmates B. The principal C. The governor D. The parent 172. What is suggested in the opening line? June 13, 1986 - they came from all over America - 200,000 heroes strong, with their families. A. The writer holds great admiration for the veterans B. The writer is opposed to the Vietnam War C. The writer was a veteran of the war D. The writer is a flag-waving patriot 173. A readability mismatch happens when the reading levels of books exceed the reading levels of the students. In this situation, the students experience frustration and they fall short of the expected or desired output. A student who finds himself/ herself in such a mismatch will likely do which of the following? A. Give an intelligent critique of the selection or story read B. Write a comprehensive reaction paper regarding the selection or story read C. Present an argument that the selection or story read was not properly written D. Manifest an expected and commensurate emotional reaction to the selection or story read 174. What does this mean: Excuses are for losers… those who take responsibility for their actions are real winners in life. This tells of? A. losers often fail because they find reasons for losing B. a winner can also be a quitter C. accountability of one’s action tells of bravery D. excuses are needed to justify any failure
  • 23. 175. A couple accepted a wedding invitation. They showed pleasure in these remarks A. All guests congratulated the organizers and the couple B. Guests came in and out of the ceremonies C. The ceremonies were very impressive D. The couple felt uneasy with the priest 176. What correction should be made to this sentence? One of the theories is that the first child receives more of the parents' attention than other children so first-borns tend to be more intellectual. A. Change is to are B. Insert a comma after children C. Change parents' to parent's D. Change theories to theory's 177. Carl Sundburg wrote "Jazz Fantasia" which has for its first stanza: Drum on your drums, batter on your banjos, sob on the long cool winding saxophones. Go to it, O jazzmen. Which words illustrate alliteration? A. Batter and banjos B. Sob and winding C. Long and cool D. To and it 178. What is meant by AT SIXES AND SEVENS in this sentence? We moved into the house last week, but I'm afraid everything, is still at sixes and sevens. A. The things have not been shipped. B. In a state of confusion C. In an orderly manner D. The boxes are still intact 179. In the opening of the free trade, if world prices of a good are less than domestic prices of that same good, _________. A.domestic consumers will experience a loss of surplus. B.domestic prices will drop to the world price level. C.all domestic producers of that good will try to find another market because they can’t compete with foreign producers. D.domestic producers will increase the quantity supplied in order to crowd out the foreign produced goods. 180. It states that as the price of the commodities increase the amount of goods the consumer is willing to purchase decrease and as the price of the commodities decrease the willingness of the consumer to buy increases and other factor remain constant. A.Law of Diminishing Marginal Utility B.Law of Gravity C.Law of Supply
  • 24. D.Law of Demand 181. A deliberate attempt to recognize and transform existing agrarian system with the intention of improving the distribution of agricultural incomes and thus fostering rural development. A.Millennium Development Plan C.Water Reform B.Land Reform D.Development Goals 182. What is the process by which the productive capacity of the economy is increased over time to bring about rising levels of national output and income? A.Economic growth C.Economic development B.Industry D.Employment 183. A system whereby the determination of exchange rate is left solely to the market forces. A.Foreign exchange liberalization B.Import liberalization C.Terms of trade D.Foreign investment 184. All are possible results when a high population growth rate continues in the Third World except A.growth of slums B.spread of diseases due to poverty and poor sanitation C.not enough schools, hospitals, roads, bridges, etc. D.increased Gross National Product 185. Which of the following is the nature of power of taxation? A.It is inherent in sovereignty. B.It is legislative in nature. C.It is subject to constitutional and inherent limitations. D.All of the above 186. A kind of tax based on the rate of which decreases as the tax base or bracket increases. A.Progressive C. Regressive B.Graduated D. Proportional 187. Agrarian reform program, Philippine experience is a success. A.The statement is generally true. B.The statement is doubtful. C.The statement is untrue. D.There is no basis to conclude. 188. It is also known as the Comprehensive Agrarian Reform Law (CARL) A.Presidential Decree # 2 B.Presidential Decree # 27 C.Republic Act 6657 D.Republic Act 5766
  • 25. 189. The Cooperatives Development Program of the government is designed primarily to support the agrarian reform program. It aims to achieve a dignified existence for the small farmers free from pernicious institutional restraints and practices. A.Only the first statement is true and correct. B.Only the second statement is true and correct. C.Both statements are true and correct. D.Both statements are untrue and incorrect. 190. The words “inappropriate, illegal, irresponsible and unaware” have prefixes which are classified as__________. A. positive C. negative B. common D. neutral 191. Using contextual clues, complete the following statement. Most flowers________ in spring. A. bloom C. wither B. weaken D. die 192. Which does NOT belong? A. Foot C. Centigrade B. Kilometer D. Mile 193. Which passage is expressed effectively and without structural error/s? A. Having called the other students and they, the secretary went home. B. Having called the other students and ourselves the secretary went home. C. Having called the other students and we, the secretary went home. D. Having called the other students and us, the secretary went home 194. When I met Albert yesterday, it was the first time I_____ her since Christmas. A. have seen C. have been seen B. saw D. had seen 195. The measure of choosing well is whether or not a man likes what he__ A. has chosen C. chose B. is choosing D. choose 196. Your father is paying for your plane ticket, _? A. isn’t it C. isn’t he B. is he D. aren’t you 197. Alin ang angkang pinagmulan ng mga wika sa Pilipinas? A. Indones B. Malay C. Indones Polinesyo D. Malayo Polinesyo 198. Alin sa mga sumusunod ang salitang pambansa? A. Pinoy C. Mapagkumbaba B. Kamusta D. Nagdadalantao 199. Ang paagtuturo ng Filipino bilang pangalawang wika ay higit na magiging mabisa kung gagami
  • 26. A. pagsasaulo B. tanong-sagot C. pagkukuwento D. modelo 200. Ang pagkautal ay matatawag na sagabal sa pagsasalita. A. saykolohikal C. pisikal B. pisyolohikal D. semantiko